Vous êtes sur la page 1sur 87

Cuaderno de Apuntes

Ministerio de Educacación
Universidad de El Salvador
Programa Jóvenes Talento El Salvador
Mentor: Lic. Mynor Ademar Melara Estrada
Instrutores:
Erick Amı́lcar Muñoz Deras
Carlos Ariel Piche Cruz

Combinatoria

Nivel VI
2 de enero de 2017

1
Índice
1. Introducción a la combinatoria 4
1.1. ¿Qué es la Combinatoria? . . . . . . . . . . . . . . . . . . . . . . . . . . . . . . . . 4
1.1.1. Ejercicios. . . . . . . . . . . . . . . . . . . . . . . . . . . . . . . . . . . . . . 4
1.2. Teorı́a de conjuntos . . . . . . . . . . . . . . . . . . . . . . . . . . . . . . . . . . . 5
1.2.1. Operaciones con conjuntos . . . . . . . . . . . . . . . . . . . . . . . . . . . . 6
1.3. Principios básicos de conteo . . . . . . . . . . . . . . . . . . . . . . . . . . . . . . . 6
1.3.1. Principio de suma: . . . . . . . . . . . . . . . . . . . . . . . . . . . . . . . . 7
1.3.2. Principio de multiplicación: . . . . . . . . . . . . . . . . . . . . . . . . . . . 7
1.3.3. Ejercicios. . . . . . . . . . . . . . . . . . . . . . . . . . . . . . . . . . . . . . 7
1.4. Notación de factorial . . . . . . . . . . . . . . . . . . . . . . . . . . . . . . . . . . . 11
1.4.1. Ejercicios. . . . . . . . . . . . . . . . . . . . . . . . . . . . . . . . . . . . . . 11

2. Combinaciones 13
2.1. Modelo de conjuntos . . . . . . . . . . . . . . . . . . . . . . . . . . . . . . . . . . . 13
2.1.1. Ejercicios. . . . . . . . . . . . . . . . . . . . . . . . . . . . . . . . . . . . . . 14
2.2. Caminos y cadenas binarias . . . . . . . . . . . . . . . . . . . . . . . . . . . . . . . 16
2.2.1. Ejercicios. . . . . . . . . . . . . . . . . . . . . . . . . . . . . . . . . . . . . . 17

3. Permutaciones y Arreglos 20
3.1. Permutaciones. . . . . . . . . . . . . . . . . . . . . . . . . . . . . . . . . . . . . . . 20
3.2. Arreglos . . . . . . . . . . . . . . . . . . . . . . . . . . . . . . . . . . . . . . . . . . 20
3.3. Permutaciones con objetos indistinguibles . . . . . . . . . . . . . . . . . . . . . . . 21
3.4. Permutaciones en arreglos circulares . . . . . . . . . . . . . . . . . . . . . . . . . . . 22
3.5. Ejemplos. . . . . . . . . . . . . . . . . . . . . . . . . . . . . . . . . . . . . . . . . . 22
3.6. Ejercicios. . . . . . . . . . . . . . . . . . . . . . . . . . . . . . . . . . . . . . . . . . 23

4. Teorema del Binomio 25


4.1. El triángulo de Pascal . . . . . . . . . . . . . . . . . . . . . . . . . . . . . . . . . . 25
4.2. Teorema (Binomio de Newton): . . . . . . . . . . . . . . . . . . . . . . . . . . . . . 26
4.3. Ejercicios. . . . . . . . . . . . . . . . . . . . . . . . . . . . . . . . . . . . . . . . . . 26

5. Inclusión-exclusión y Desordenes 28
5.1. Inclusión-exclusión . . . . . . . . . . . . . . . . . . . . . . . . . . . . . . . . . . . . 28
5.1.1. Ejercicios. . . . . . . . . . . . . . . . . . . . . . . . . . . . . . . . . . . . . . 29
5.2. Desórdenes . . . . . . . . . . . . . . . . . . . . . . . . . . . . . . . . . . . . . . . . 30
5.2.1. Ejercicios. . . . . . . . . . . . . . . . . . . . . . . . . . . . . . . . . . . . . . 31

6. Separadores 33
6.1. Ejemplos. . . . . . . . . . . . . . . . . . . . . . . . . . . . . . . . . . . . . . . . . . 33
6.2. Ejercicios. . . . . . . . . . . . . . . . . . . . . . . . . . . . . . . . . . . . . . . . . . 34

7. Comparaciones 37
7.1. Ejercicios. . . . . . . . . . . . . . . . . . . . . . . . . . . . . . . . . . . . . . . . . . 37

8. Principio de Casillas 38
8.1. Ejercicios. . . . . . . . . . . . . . . . . . . . . . . . . . . . . . . . . . . . . . . . . . 39

2
9. Coloración y Paridad 43
9.1. Coloración. . . . . . . . . . . . . . . . . . . . . . . . . . . . . . . . . . . . . . . . . 43
9.1.1. Ejercicios. . . . . . . . . . . . . . . . . . . . . . . . . . . . . . . . . . . . . . 44
9.2. Problemas dinámicos . . . . . . . . . . . . . . . . . . . . . . . . . . . . . . . . . . . 46
9.2.1. Ejercicios. . . . . . . . . . . . . . . . . . . . . . . . . . . . . . . . . . . . . . 46

10.Recurencia 48
10.1. Ejemplos. . . . . . . . . . . . . . . . . . . . . . . . . . . . . . . . . . . . . . . . . . 48
10.2. Ejercicios. . . . . . . . . . . . . . . . . . . . . . . . . . . . . . . . . . . . . . . . . . 49

3
Capı́tulo 1
1. Introducción a la combinatoria
1.1. ¿Qué es la Combinatoria?
Si se pregunta qué es la Combinatoria, podrı́a responderse algo ası́ como “el arte y ciencia de
contar”. Pero si bien los métodos de recuento forman parte esencial de la Combinatoria, esta con-
templa también otros aspectos, entre ellos: el estudio de configuraciones conocidas, el estudio de
la existencia de ciertas configuraciones, el conteo del número de configuraciones de un tipo dado,
la enumeración o descripción de configuraciones, la optimizacón combinatoria, es decir la determi-
nación de las configuraciones que maximizan o minimizan una función dada, y otras. En cualquier
caso el campo abierto a la Combinatoria es amplio y fascinante, repleto de bellos resultados e
interesantes problemas abiertos.

1.1.1. Ejercicios.
Indicaciones: Los problemas siguientes te servirán para comprender las técnicas básicas de con-
teo, estás en libertad de solucionarlos de la manera que creas conveniente, ¡vamos inténtalo!

1. ¿De cuántas maneras podemos ir de una ciudad A a una ciudad C pasando por la ciudad B,
si hay tres caminos de A a B y cuatro caminos de B a C? (No se vale regresarse)

2. ¿De cuántas maneras podemos ir de una ciudad A a una ciudad C, si hay tres caminos de A
a B y cuatro caminos de B a C y además hay cinco caminos directos entre A y C? (No se
vale regresarse)

3. Cuatro chicos(as) son enviados al concurso de declamación del colegio. Para esperar su turno,
tienen que alinearse en fila en la entrada del auditorio. ¡Todos quieren ser el primero, desde
luego! Supongamos que los niños(as) se llaman Andrea, Benito, Carmen y Daniel (los llama-
remos A, B, C y D). Queremos escribir todos los órdenes posibles en que podrı́an alinearse.
Por ejemplo: si escribimos A B C D, significa Andrea primero, Benito segundo, Carmen
tercero y Daniel cuarto, ¿Cuántas y cuáles formas diferentes hay en total?

4. Disponemos de tres cartas iguales. Se dispone de cinco sobres de diferentes colores: amarillo,
blanco, crema, dorado y escarlata, para guardarlas. Si cada sobre sólo puede contener, a lo
sumo, una carta. ¿De cuántas formas podemos colocar las tres cartas en los cinco sobres
diferentes? Ejemplo: podemos colocar una carta en el sobre amarillo, otra en el blanco y otra
en el crema.

5. El garaje de la casa de Ángel tiene cinco plazas. Con Ángel viven sus hermanas Beatriz y
Carmen; cada uno tiene coche que pueden colocar cada dı́a en el lugar que prefieran, si no
está ocupado. Este es el esquema de la cochera:
1 2 3 4 5

4
Por ejemplo, Ángel puede estacionar su coche en el estacionamiento número 1, Beatriz en el
número 2 y Carmen en el número 4. ¿De cuántas formas pueden Ángel, Beatriz y Carmen
estacionar sus coches en la cochera?

6. Manuel tiene cuatro coches de colores diferentes (azul, blanco, verde y rojo) y decide re-
galárselos a sus hermanos Luis y Karen. ¿De cuántas formas diferentes puede regalar los
coches a sus hermanos? Ejemplo: podrı́a dar los cuatro coches a su hermano Luis.

7. Un tetraedro regular es un poliedro con cuatro caras que son triángulos equiláteros congruen-
tes. ¿De cuántas formas podemos colorearlo con dos colores? Dos coloraciones son iguales si
podemos rotar un tetraedro coloreado de una forma, de manera que se vea como la segunda
coloración. No necesariamente deben usarse los dos colores.

8. En una fiesta, cuatro amigos se van a dar regalos entre sı́, de manera que cada uno de un
regalo y reciba otro, desde luego nadie debe darse el regalo a sı́ mismo (algo ası́ como si
jugaran Amigo Secreto). ¿De cuántas formas es posible hacer la distribución?

1.2. Teorı́a de conjuntos


Con un conjunto nos referimos a una “colección bien definida de elementos, dentro de un
todo”. Los elementos de un conjunto pueden ser números, personas, letras,..., prácticamente cual-
quier cosa, incluso conjuntos. Al escribir S = {a, b, c} estamos diciendo que S es el conjunto que
contiene los elementos a, b y c. Para indicar que a es un elemento de S se escribe a ∈ S. En un
conjunto los elementos no se repiten (solo pueden estar o no estar en el conjunto) y además no
tienen un orden, es decir que si se hace un listado de los elementos, el orden no es importante.
Dos conjuntos son iguales si y solo si, tienen los mismos elementos, por ejemplo S = {a, b, c} y
T = {c, b, a, b, c, c} son ambos el mismo conjunto. Un conjunto puede ser definido enunciando una
propiedad que permita determinar si un objeto pertenece o no a ese conjunto, por ejemplo, puede
definirse el conjunto A como “El conjunto de los números naturales que son primos” y en tal caso
serı́a A = {2, 3, 5, 7, 11, 13, 17, 19, . . . }.

Si un conjunto “A” cumple que todos sus elementos son también elementos de un conjunto B,
se dice que A es subconjunto de B o que A está incluido en B y se escribe A ⊂ B. Por ejemplo,
el conjunto de los números naturales es un subconjunto del conjunto de los números racionales.
El conjunto que no tiene elementos se llama conjunto vacı́o y se representa con la letra griega Φ
o escribiendo dos llaves sin algo entre ellas: { }. Además dado un conjunto A, puede pensarse en
el conjunto tal que sus elementos son todos los subconjuntos del conjunto A, a este conjunto le
llamaremos el conjunto potencia de A y se denotará por P (A). El conjunto vacı́o se considera que
está incluido en cualquier conjunto.

En realidad no es tan fácil definir exactamente qué es un conjunto. Al decir que “es parte de
un todo y debe estar bien definido” nos referimos a que debe poder determinarse si un objeto
pertenece o no al conjunto y que dicho conjunto no es tan “grande” como para dar problemas.
Este detalle es bastante técnico, pero por ejemplo, no puede pensarse en “el conjunto de todas las
cosas” como un conjunto, porque entonces su conjunto potencia tendrı́a más elementos y no tiene
sentido que haya un conjunto más grande que el que tiene todas las cosas. Con los conjuntos que
trabajaremos no sucederán este tipo de problemas, aunque algunos de ellos sean infinitos, pero es

5
importante notar que no toda colección de cosas es un conjunto.

Cardinalidad de un conjunto: A la cantidad de elementos que contiene un conjunto se le


conoce como la cardinalidad del conjunto. Por ejemplo, si es A = {a, b, c, d, e} se tiene que la
cardinalidad del conjunto A es 5 y se denota escribiendo el conjunto entre dos barras verticales,
ası́: |A| = 5. También se puede escribir Car(A).

1.2.1. Operaciones con conjuntos


Unión de conjuntos: Sean A y B dos conjuntos, el conjunto
unión de A con B, que se denota por A ∪ B, está formado por
todos los elementos que pertenecen a A o a B, es decir, por los
elementos que están solo en el conjunto A o están solo en B o
están en ambos conjuntos.

Por ejemplo, si A = {0, 1, 2, 7, 8} y B = {1, 3, 4, 5, 8} entonces se


tiene que A ∪ B = {0, 1, 2, 3, 4, 5, 7, 8}.

Intersección de conjuntos: Sean A y B dos conjuntos, el


conjunto intersección de A con B, que se denota por A ∩ B, es el
conjunto formado por todos los elementos que pertenecen a A y a B.

Por ejemplo, si A = {0, 1, 2, 7, 8} y B = {1, 3, 4, 5, 8} entonces se


tiene que A ∩ B = {1, 8}.

Diferencia de conjuntos: Sean A y B dos conjuntos, se denota


por A − B, al conjunto formado por todos los elementos que
pertenecen a A pero no a B.

Por ejemplo, si A = {0, 1, 2, 7, 8} y B = {1, 3, 4, 5, 8} entonces se


tiene que A − B = {0, 2, 7}.

Complemento de un conjunto: Sean A y U dos conjuntos tales


que A ⊂ U , entonces el complemento del conjunto A en el conjunto
U , que se denota por AC , es el conjunto formado por todos los
elementos de U que no están en A.

Por ejemplo, si A = {0, 1, 2, 7, 8} y U = {0, 1, 2, 3, 4, 5, 6, 7, 8, 9}


entonces se tiene que AC = {3, 4, 5, 6, 9}.

1.3. Principios básicos de conteo


Problema 1: Se dispone de un grupo de 3 mujeres y 4 hombres.
a) ¿De cuántas maneras se puede seleccionar una pareja de personas del mismo sexo?
b) ¿De cuántas maneras se puede seleccionar un grupo de 4 personas compuesto por dos mujeres
y dos hombres?

6
Solución: Cómo las mujeres y los hombres son todos diferentes entre sı́, pueden utilizarse las
letras a, b, c para representar a las tres mujeres y las letras x, y, z, w para representar a los hombres.

Luego, para el literal a) hay dos opciones: que la pareja la formen dos mujeres o que la formen
dos hombres. Hay 3 parejas formadas por mujeres, las cuales son ab, ac y bc, y hay 6 parejas
formadas por hombres, son xy, xz, xw, yz, yw y zw. Por lo tanto, el total de maneras de formar
una pareja del mismo sexo es 3+6=9.

Para el literal b) hay que notar que al tomar una pareja de mujeres y agregar cualquiera de las
parejas de hombres se puede formar un grupo de 4 personas como los que se pide, y que de hecho,
cualquiera de esos grupos de 4 se puede formar ası́.

abxy acxy bcxy


abxz acxz bcxz
abxw acxw bcxw
abyz acyz bcyz
abyw acyw bcyw
abzw aczw bczw
Por lo tanto, el número de maneras diferentes de formar un grupo de 4 personas, dos de las
cuales sean mujeres y dos sean hombres es 3(6)=18. Hay que observar que por cada una de las 3
parejas de mujeres hay 6 parejas de hombres con las que se puede completar el grupo.
En este problema se muestran los dos principios básicos de conteo siguientes:

1.3.1. Principio de suma:


Si un evento A puede ocurrir de m maneras, un evento B puede ocurrir de n maneras y los
eventos son mutuamente excluyentes, entonces el total de maneras en que puede ocurrir o el evento
A o el evento B (es decir, uno de los dos eventos) es m + n.

1.3.2. Principio de multiplicación:


Si un evento A puede ocurrir de m maneras y un evento B puede ocurrir de n maneras, entonces
el número de maneras en que puede ocurrir el evento A y el evento B es mn.

Nótese que en el literal a) del problema 1, los eventos A: “seleccionar una pareja de mujeres”
y B: “seleccionar una pareja de hombres”, son opciones o casos y debı́a ocurrir uno y solo uno
de ellos, es decir, al formar una pareja habı́a que decidir si iba a estar compuesta por dos muje-
res o por dos hombres, mientras que en el literal b), dichos eventos A y B son pasos que deben
hacerse (ambos) para construir un grupo con las condiciones pedidas (en la solución se formó pri-
mero la pareja de mujeres y luego se completó con alguna manera de formar la pareja de hombres,
si se hubiera seleccionado primero la pareja de hombres, se hubiera obtenido los mismos resultados).

1.3.3. Ejercicios.
Indicaciones: Resuelva en forma clara y ordenada cada uno de los problemas que se le pre-

7
sentan, dejando constancia de sus soluciones

1. ¿Cuántos números enteros hay

a) desde 352 hasta 1756?


b) entre 217 y 334 inclusive?
c) entre 217 y 334?
d ) mayores que 217 y menores o iguales a 334?

2. El mayor de r enteros consecutivos es k. ¿Cuál es el menor?

3. Sean los conjuntos A = {a, b, c, d, e} y B = {0, 1, 2, 3, 4, 5, 6, 7, 8, 9}.

a) ¿De cuántas formas es posible seleccionar un elemento del conjunto A y uno del B?
b) ¿De cuántas formas es posible seleccionar un elemento que puede ser del conjunto A o
del B?

4. Una compañı́a que recibe pedidos de compra por correo, ofrece 23 tipos de sandalia para
damas. Si cada modelo está disponible en doce medidas de largo, tres de ancho y seis colores
distintos, ¿cuántas clases distintas de sandalias deberá mantener en sus almacenes?.

5. Vı́ctor desea viajar de San Salvador a San José y tiene a su disposición 3 lı́neas aéreas directas
y 5 lı́neas terrestres también directas. ¿de cuántas maneras diferentes puede realizar el viaje?

6. Se dispone de 5 niños y 3 niñas para formar una pareja de baile. ¿De cuántas maneras
diferentes puede formarse la pareja?

7. Rosa posee 3 blusas distintas, 2 pantalones diferentes y 4 pares de zapatos diferentes. ¿De
cuántas maneras distintas puede vestirse utilizando las prendas mencionadas (una de cada
clase)?

8. En una tienda venden tres clases de cubiertos: cucharas, tenedores y cuchillos. Además, hay
5 tipos de cuchara, 6 tipos de tenedor y 4 tipos de cuchillos. ¿De cuántas maneras podemos
comprar dos cubiertos de distinta clase?

9. ¿Cuántas comisiones integradas por un chico y una chica pueden formarse con 5 chicos y 8
chicas, si cierto chico rehúsa trabajar con dos chicas en particular?

10. ¿Cuántos pares ordenados de enteros (x, y) hay tales que 0 < |xy| ≤ 5? (Las barras |·| indican
valor absoluto)

11. Diremos que un par ordenado de enteros positivos es centenario si cada número sumado
con los dı́gitos del otro número suman 100. Por ejemplo (83, 89) es un par centenario pues:
83 + 8 + 9 = 89 + 8 + 3 = 100 Determinar la cantidad de pares centenarios que existen.
Observa que (83,89) es distinto de (89,83).

12. ¿Cuántos cuadrados hay en una cuadrı́cula de 8×8 tales que sus lados sean paralelos a los
bordes de la cuadrı́cula? ¿Cuántos cuadrados con esas condiciones hay en una cuadrı́cula de
n × n?

8
13. En cada caja hay tantos chocolates como cajas hay en cada cuarto, y hay tantas cajas en
cada cuarto como cuartos hay en cada casa, y hay tantos cuartos en cada casa como casas
hay en una cuadra. Hay 81 chocolates en una cuadra. ¿Cuántos chocolates hay en cada caja?

14. ¿Cuántos números de tres dı́gitos pueden formarse utilizando los elementos del conjunto
{0, 1, 2, 3, 4}? ¿Cuántos de ellos no tienen dı́gitos repetidos? ¿Cuántos no tienen dı́gitos re-
petidos y son pares?

15. ¿Cuántos enteros positivos de cuatro dı́gitos pueden ser formados con los elementos del
conjunto de los dı́gitos {0, 1, 2, 3, 4, 5} tal que no haya dı́gitos repetidos y el entero resultante
sea múltiplo de 3?

16. ¿De cuántas maneras se pueden acomodar 4 alumnos(as) en una fila de 5 asientos si dos de
ellos deben estar juntos?

17. ¿De cuántas maneras diferentes se puede distribuir cuatro camisas de diferente color en tres
cajones distintos?

18. Cada cuadro de una cuadrı́cula (fija) de 5×6 se colorea de blanco o de negro. ¿Cuántas
coloraciones diferentes de la cuadrı́cula existen?

19. Hay que colocar a 5 hombres y 4 mujeres en una fila de modo que las mujeres ocupen los
lugares pares. ¿De cuántas maneras puede hacerse?

20. Mozart compuso un vals con 11 posibilidades distintas para 14 de los 16 compases y 2
posibilidades para cada uno de los restantes. ¿Se habrán llegado a escuchar alguna vez todas
las realizaciones posibles?

21. Ana y Marı́a vieron a dos hombres alejarse en automóvil frente a una joyerı́a, justo antes
de que sonara una alarma contra robos. Cuando fueron interrogadas por la policı́a, las dos
jóvenes dieron la siguiente información acerca de la placa (que constaba de dos letras seguidas
de cuatro dı́gitos). Marı́a estaba segura de que la segunda letra de la placa era una O o una
Q, y que el último dı́gito era un 3 o un 8. Ana dijo que la primera letra de la placa era
una C o una G y que el primer dı́gito era definitivamente un 7. ¿Cuántas placas diferentes
tendrá que verificar la policı́a?

22. ¿Cuántas maneras distintas hay de colocar una torre blanca y una negra en un tablero de
ajedrez de modo que

a) se ataquen entre sı́?


b) no se ataquen entre sı́?
c) ¿Cuál serı́a la solución de a) y b) si las dos torres fueran negras?

23. ¿Cuántas maneras distintas hay de colocar un caballo blanco y uno negro en un tablero de
ajedrez de modo que no se ataquen entre sı́?

24. Encuentra el número de maneras de poner 3 torres distintas en un tablero de 5×5, sin que
cualesquiera dos de ellas se ataquen. ¿Y si fueran n torres distintas en un tablero de n × n?

25. ¿Cuántos números de seis dı́gitos tienen al menos un dı́gito par?

9
26. ¿Cuántos números de cinco dı́gitos tienen al menos dos dı́gitos iguales?

27. ¿Cuántos enteros del 1 al 1000000 tienen al menos un 1 entre sus cifras?

28. ¿De cuántas formas se pueden sentar cinco personas en fila, si dos de ellas nunca deben estar
juntas?

29. Se dispone de 6 cajas de colores diferentes: rojo, azul, amarillo, blanco, negro y celeste. Si
se debe formar una columna colocando una encima de otra, ¿de cuántas maneras se puede
apilar las 6 cajas con la condición de que la caja amarilla no quede en la base?

30. Se desea elegir un presidente, un tesorero y un secretario en un comité integrado por diez
personas. ¿De cuántas maneras es posible hacer esto?

31. ¿De cuántas maneras podemos elegir un portero, un capitán y un capitán suplente en un
equipo de fútbol de once personas? (el portero puede ser capitán o capitán suplente, pero el
capitán y el capitán suplente deben ser personas distintas).

32. En Braille se utiliza como base un rectángulo como el de la figura, en el que en cada casilla
se puede colocar un punto en relieve o dejarla vacı́a. Si cada combinación de puntos y vacı́os
en un rectángulo representa un carácter, ¿cuántos caracteres se pueden representar? (un
carácter es un sı́mbolo, como una letra o un número)

33. En un teatro hay 20 ventiladores y para mantener el aire puro se desea que siempre esté al
menos un ventilador encendido. Si en un momento determinado se anota en una tabla la
condición de apagado o encendido de cada ventilador, ¿cuántos registros diferentes, que
cumplan con lo que se desea, es posible obtener?

34. Si p, q y r son tres números primos distintos, ¿cuántos y cuáles son los divisores del número
N = p2 qr2 ? (considera que un divisor de N debe ser de la forma pa q b rc , analiza los valores
que pueden tomar a, b y c.

35. ¿Cuántos subconjuntos del conjunto {1, 2, 3, ..., 10} existen tales que la diferencia entre
cualesquiera dos de sus elementos sea mayor o igual que 3?

36. Halla todos los pares de números naturales x, y con x < y tales que la suma de todos los
números naturales que están estrictamente entre ambos es 1999.

¡Para divertirse!...(Optimización combinatoria)

1. El señor Asamantecas tiene un asador pequeño, donde apenas caben dos chuletas. Su mujer
y su hija Clara se mueren de hambre y están ansiosas por comer cuanto antes. El problema
es asar las tres chuletas en el mı́nimo tiempo posible.

Sr. Asamantecas: Vamos a ver, hacen falta 20 minutos para asar una chuleta por los dos
lados, pues cada uno tarda 10. Como puedo preparar dos chuletas a la vez, en 20 minutos

10
puedo tener listas dos. La tercera tardará otros 20 minutos. Ası́ que la comida estará a punto
dentro de 40 minutos.

Clara: ¡Pero papá! ¡Si puedes hacerlo en mucho menos! Acaba de ocurrı́rseme cómo ahorrar
10 minutos. ¿Cuál fue la feliz idea que se le ocurrió a Clara?

2. ¿Cómo harı́as para traer de un rı́o seis litros de agua, si no tienes a tu disposición, para medir
el agua, mas que dos recipientes, uno de cuatro litros y otro de nueve?

3. Un pastor tiene que pasar un lobo, un conejo y una col de una orilla de un rı́o a la otra orilla.
Dispone de una barca en la que sólo caben él y una de las tres cosas anteriores. Si deja solos
al conejo y al lobo, éste se come a aquél; si deja al conejo con la col, aquél se la come. ¿Cómo
debe proceder para llevar las tres cosas a la orilla opuesta?

4. Hay cuatro botes en una de las orillas de un rı́o; sus nombres son Ocho, Cuatro, Dos y Uno,
porque esa es la cantidad de horas que se tarda cada uno de ellos en cruzar el rı́o. Se puede
amarrar un bote a otro pero no más de uno, y entonces el tiempo que tardan en cruzar es
igual al más lento de los dos botes. Un sólo marinero debe de llevar todos los botes a la otra
orilla. Es decir, que un bote no puede cruzar sin marinero y el marinero viaja con uno o
dos botes. ¿Cuál es la menor cantidad de tiempo que necesita el marinero para completar el
traslado?

1.4. Notación de factorial


Factorial de un número entero positivo

Se llama factorial de un número n ∈ Z+ al producto de los n factores consecutivos desde 1


hasta n y se denota por n!. Ası́:

n! = 1(2)(3)...(n − 2)(n − 1)(n)

Por ejemplo:
4! = (4)(3)(2)(1)
= 24

También:
4! = (4)(3!)
= (4)(3)(2!)

Y en general: n! = n(n − 1)!


= n(n − 1)(n − 2)!
= n(n − 1)(n − 2)(n − 3)!
= n(n − 1)(n − 2)(n − 3)...(n − r)!; r<n

1.4.1. Ejercicios.
1. Efectúa las siguientes operaciones:

a) 5! =

11
b) 7! =
c) (8 − 5)! =
d ) 8! − 5! =
e) 4!(5!) =
f ) 7(8!) − 6! =

2. Simplifica las expresiones siguientes:


 
8! 8 8! 2013!
a) b) ! c) d)
4! 4 4 2013
7! 9! 12! 4(6!)
e) f) g) h)
(7 − 2)! 3!(9 − 3)! 3!(4!)(5!) 6!(4!)(6)

3. Halla el valor de x sabiendo que:

a) x! = 110(x − 2)!
b) 12x! + 5(x + 1)! = (x + 2)!

4. ¿De cuántas formas diferentes se pueden ordenar las letras de la palabra IMPUREZA?

5. ¿De cuántas formas se pueden colocar 5 libros diferentes en fila en un mostrador?

12
Capı́tulo 2
2. Combinaciones
2.1. Modelo de conjuntos
Problema 1: Se dispone de un grupo de 3 mujeres y 4 hombres.

a) ¿De cuántas maneras se puede seleccionar una pareja de personas del mismo sexo?

b) ¿De cuántas maneras se puede seleccionar un grupo de 4 personas compuesto por dos mujeres
y dos hombres?

En el problema 1 se ha presentado la situación de que a partir de un grupo de objetos dispo-


nibles se debe seleccionar algunos de ellos. En general, si se dispone de n objetos distintos y se
desea seleccionar r de ellos (con 0 ≤ r ≤ n), entonces el número de maneras distintas en que puede
hacerse es denotado por Cnr .

Ası́ por ejemplo, el total de formas en que se pueden seleccionar tres vocales del afabeto caste-
llano, es decir tres elementos del conjunto {a,e,i,o,u} es el número C53 . Es importante comprender
que no se está tomando en cuenta el orden en que se seleccionan los objetos, solo importa cuáles
son los objetos seleccionados.

Con notación de números combinatorios, las soluciones del problema 1 son a) C32 + C42 y b)
(C32 ) (C42 ).

Los números combinatorios también se denotan por nr , nCr o Crn , ası́ que al leer un libro o


documento sobre combinaciones, hay que tener claro cuál es la notación que se está utilizando.

Problema 2: Se dispone de los números 1, 2, 3, 4, 5 y 6 para llenar tres casillas en blanco,


escribiendo un número en cada casilla y tal que los tres números utilizados sean distintos entre sı́.

a) ¿De cuántas formas se puede hacer?

b) Si al llenar las casillas, los números seleccionados se ubican en orden creciente, ¿de cuántas
maneras se puede hacer?

Solución:

a) El problema consiste en seleccionar ordenadamente tres de los seis números disponibles, la


solución se obtiene calculando el producto

6(5)(4) = 120

dado que hay 6 maneras de seleccionar el primer número, por cada una de ellas hay 5 maneras
de seleccionar el segundo número y finalmente hay 4 maneras de seleccionar el tercer número.
Ası́ que hay 120 maneras distintas de llenar las casillas utilizando tres números diferentes, de
los seis disponibles.

13
b) En este caso, lo único que importa son los números que se seleccionan y no el orden en que se
seleccionan o se ubican, puesto que una vez elegidos los tres números, la forma de ubicarlos es
única, ası́ que la solución es C63 .

Por ejemplo, si los números seleccionados son 1, 4 y 6, en el literal a) esos tres números se
podı́an ubicar de 3!=6 maneras: 146, 164, 416, 461, 614, 641; pero para el literal b) la única
manera de ubicarlos es en el orden 146. La solución al literal b) puede obtenerse entonces a
partir de la solución del literal a) observando que de los 120 arreglos que corresponden a selec-
cionar y ubicar ordenadamente a tres de los seis números disponibles, se pueden formar grupos
de seis arreglos en los que intervienen los mismos tres números y cada uno de estos grupos de
seis arreglos del literal a) corresponde a una manera de ubicar los números para el literal b).
6(5)(4) 120
Por lo tanto hay = = 20 maneras de ubicar en las casillas tres números diferentes,
3! 6
de los seis disponibles, en orden creciente. Se tiene que C63 = 20.

A partir del literal b) del problema anterior puede deducirse que el total de combinaciones
tomando r objetos de n disponibles, que se representa por Cnr , viene dado por la expresión:
n(n − 1)(n − 2) . . . (n − r + 1)
Cnr =
r!
n!
(n − r)!
=
r!
n!
=
r!(n − r)!

2.1.1. Ejercicios.
1. ¿Cuántos diferentes licuados mezclando dos frutas pueden hacerse si se dispone de fresas,
uvas, bananos, coco, piña, zapote y melón? ¿cuántos con tres frutas?

2. Se tiene el conjunto {a, b, c, d, e}. ¿Cuántos y cuáles de sus subconjuntos tienen una letra?
¿y dos letras? ¿y tres letras? ¿y cuatro letras? ¿y cinco letras? ¿y ninguna letra?

3. ¿Cuántos subconjuntos tiene el conjunto {a, b, c, d, e}?

4. De un grupo de seis hombres y cuatro mujeres se desea formar una comisión de tres personas,
¿cuántas comisiones distintas se pueden formar si

a) no hay más restricciones.


b) debe haber un hombre y dos mujeres?
c) debe haber exactamente un hombre?
d ) debe haber al menos un hombre?

5. Se va a seleccionar un comité de 12 personas de un grupo de 10 hombres y 10 mujeres. ¿De


cuántas formas distintas se puede hacer la selección si:

a) no hay restricciones?

14
b) debe haber 6 hombres y 6 mujeres?
c) debe haber un número par de mujeres?
d ) debe haber más mujeres que hombres?
e) debe haber 8 hombres como mı́nimo?

6. Si se dispone de cinco puntos en el plano, de los cuales no hay tres alineados, ¿cuántos
segmentos de recta, que unan dos de dichos puntos se pueden trazar?

7. ¿Cuántas diagonales tiene un pentágono? ¿cuántas diagonales tiene un polı́gono convexo de


n lados?

8. En una bolsa hay 5 pelotas rojas y 9 azules y se quiere formar una fila con todas ellas. ¿De
cuántas maneras distintas puede quedar la fila? ¿De cuántas maneras si no deben aparecer
dos bolas rojas consecutivas?

9. ¿Cuántas cadenas de 4 ceros y 3 unos pueden formarse? (diremos que una cadena como ésta
es de longitud 7 por estar compuesta de 7 elementos)

10. ¿Cuántas cadenas de longitud n están compuestas por k unos y n − k ceros?

11. ¿Cuántas cadenas de longitud 6 contienen como máximo 3 unos?

12. De un grupo de 24 personas se quieren elegir 5 representantes de la siguiente forma: Manuel


y Luis deben estar en el grupo elegido. Hay 8 mujeres en total pero a lo más deben figurar
2 en el grupo. ¿De cuántas maneras distintas puede hacerse la elección?

13. Un grupo de 6 alumnos(as) quiere dividirse en 3 equipos de 2 personas cada uno. ¿De cuántas
y cuáles formas puede hacerse la distribución si

a) un grupo discutirá sobre el valor de la cooperación, otro sobre la solidaridad y otro


sobre la honestidad?
b) los tres grupos deben discutir sobre el valor de la honestidad?

14. Demuestra la identidad del presidente n+1


 n+1 n
k+1
= k+1 k contando de dos maneras diferentes
las formas en que teniendo n+1 personas disponibles se seleccionen k +1 de ellas para formar
un comité y que una de esas k + 1 personas sea designada como presidente del comité.

15. Demuestra que si p es un número primo y 0 < k < p entonces kp es múltiplo de p.




16. En un tablero de 5×5 tratamos de contar todos los posibles rectángulos cuyos lados son para-
lelos a los bordes del tablero. Dos rectángulos se considerarán diferentes si son diferentes sus
dimensiones o las posiciones que ocupan son diferentes. ¿Cuántos rectángulos son? ¿Cuántos
rectángulos serı́an si el tablero fuera de n × n?

17. Encuentra el número de maneras de poner 3 torres idénticas en un tablero de 5×5, sin que
cualesquiera dos de ellas se ataquen.

18. ¿Cuántas colecciones de tres números enteros del 1 al 25 suman un múltiplo de cinco? (Un
ejemplo de tales colecciones es {1, 11, 23}).

15
19. ¿Cuántos números de tres dı́gitos abc (con a 6= 0) son tales que a + 3b + c es múltiplo de 3?

20. En cierto paı́s hay varios aeropuertos. Una aerolı́nea ofrece diariamente vuelos directos que
conectan cualesquiera dos aeropuertos. Cada dı́a la aerolı́nea realiza 30 vuelos. ¿Cuántos
aeropuertos hay?

2.2. Caminos y cadenas binarias


Suponga la siguiente situación, una persona se encuentra de paseo en una ciudad y está in-
teresada en ir de la esquina A hasta la esquina B por la ruta más corta posible, la ciudad tiene la
caracterı́stica de tener calles formando una cuadrı́cula como lo ilustra la siguiente figura.

Figura 1: Esquema de calles de una ciudad

¿De cuántas formas podemos realizar dicho recorrido?

En primer lugar hay que notar que los caminos de longitud mı́nima, deben estar formados por
10 movimientos a la derecha y 6 hacia arriba, ası́ que cada camino puede asociarse a un arreglo de
10
10 sı́mbolos → y 6 sı́mbolos ↑ , lo cual puede hacerse de C16 maneras, dado que solo se necesita
seleccionar la posición de los 10 sı́mbolos →. En general dada una retı́cula de dimensión k ×(n−k),
como la siguiente:

Figura 2: Cuadrı́cula de dimensiones k × (n − k)

La cantidad de caminos de longitud mı́nima que llevan del punto A al punto B, viene dada por
el número combinatorio
k
Ck+n−k = Cnk .

16
2.2.1. Ejercicios.
Indicaciones: Resuelva en forma clara y ordenada cada uno de los problemas que se le pre-
sentan, dejando constancia de sus soluciones

1. Para la siguiente cuadrı́cula, cuántos caminos de longitud mı́nima:

a) Llevan de A a M.
b) Llevan de M a B.
c) Llevan de A a B pasando por M.
d ) Llevan de A a B pasando por N.
e) Llevan de A a B pasando por M y N.
f ) Llevan de A a B pasando por M o N.
g) Llevan de A a B.
h) Llevan de A a B y no pasan por M ni por N.

Figura 3

2. Para la siguiente cuadrı́cula, cuántos caminos formados solo por movimientos hacia la derecha
o hacia arriba:

a) Llevan de A a B.
b) Llevan de A a M.
c) Llevan de A a B pasando por M.
d ) Llevan de A a B pero no pasan por M.
e) Llevan de A a B pasando por N.
f ) Llevan de A a B pasando por M y N.
g) Llevan de A a B pasando por M o N.

Figura 4

17
3. En la cuadrı́cula del ejercicio anterior, ¿cuántos caminos de longitud 7, iniciando en A, están
formados solo por movimientos hacia arriba o hacia la derecha?

4. En una cuadrı́cula de n × n , ¿cuántos caminos de longitud n , iniciando en el vértice inferior


izquierdo, están formados solo por movimientos hacia arriba o hacia la derecha?

5. Demuestra con un argumento combinatorio que 2n = no + n1 + n2 + ... + nn .


   

6. ¿Cuántos subconjuntos tiene un conjunto de n elementos?

7. Explica utilizando caminos porque es cierta la propiedad simétrica:


   
n n
= .
k n−k

8. Usar caminos para probar la propiedad recursiva de Pascal:


     
n+1 n n
= +
k+1 k k+1

para 0 ≤ k < n.

9. Demuestra que si n es un entero positivo, entonces


   2  2  2  2
2n n n n n
= + + + ... + .
n o 1 2 n

Debes demostrarlo de dos maneras: con un argumento contando caminos en una cuadrı́cula
de n × n y con un argumento que consista en contar de dos maneras diferentes las formas de
seleccionar n objetos o personas de un grupo de 2n disponibles.

10. Sea p un número primo, demuestra que 2p



p
− 2 es múltiplo de p2 .

18
11. Para la siguiente cuadrı́cula, cuántos caminos de longitud mı́nima:

a) Llevan de A a B.
b) Llevan de A a B pasando por M.
c) Llevan de A a B pasando por N.
d ) Llevan de A a B pasando por M y N.
e) Llevan de A a B pasando por M o N.
f ) Llevan de A a B y no pasan por M ni por N.
g) Llevan de A a B y no pasan por los dos puntos M y N.
h) Llevan de A a B sin pasar por alguno de los puntos de la última lı́nea vertical (excepto
B, claramente).
i ) Llevan de A a B pasando por un solo punto de la segunda lı́nea horizontal.

Figura 5

19
Capı́tulo 3
3. Permutaciones y Arreglos
3.1. Permutaciones.
Ejemplo 1 ¿De cuántas formas pueden ordenarse los elementos del conjunto A = {a1 , a2 , a3 , ..., an }?
(Todos los n elementos de A son distintos entre sı́).

Cada forma de ordenar los elementos de A diremos que es una permutación de los elementos de
A. Observe que para contar los posibles ordenamientos nos basta definir el elemento que ocupará la
primera posición, el que ocupará la segunda, el de la tercera posición y ası́ hasta el que ocupará la
n-ésima posición. En este caso, la primera posición puede ser ocupada por uno cualquiera de los
elementos de A, es decir que se cuenta con n posibilidades; para la segunda posición ya sólo dispo-
nemos de n − 1 posibilidades y para la posición tercera sólo hay n − 2 posibilidades y se continúa
de esta manera hasta que para la n-ésima posición solo quedará un elemento disponible. Por tanto
existen n(n − 1)(n − 2)...(2)(1) = n! maneras de ordenar los elementos del conjunto A, es decir,
hay n! permutaciones de n elementos.

Ejemplo 2 ¿Cuántas cadenas de longitud 7 se pueden formar usando los elementos del conjunto
B = {a, e, i, o, u}?

En este caso no se están ordenando los elementos del conjunto B, pero si se deben ir seleccionando,
en forma ordenada, elementos de B para construir la cadena. Ası́, para elegir el primer elemento
de la cadena hay 5 opciones, para elegir el segundo elemento de la cadena también hay 5 opciones
porque las letras se pueden repetir y de igual manera ocurre para llenar los espacios restantes, por
lo tanto el total de cadenas de longitud 7 que se pueden formar usando los elementos de B es 57 .

3.2. Arreglos
Ejemplo 3 Se dispone de los dı́gitos del 3 al 9 y se desea formar números de tres cifras distintas.
¿Cuántos números con esas condiciones se pueden formar?

Para elegir la primera cifra, se dispone de 7 dı́gitos, para elegir la segunda cifra se dispondrá solo
de 6 dı́gitos y para elegir la tercera cifra se dispondrá solo de 5 dı́gitos, por lo tanto, aplicando
el principio de multiplicación se tiene que la cantidad de números de tres cifras distintas que se
pueden formar utilizando los dı́gitos del 3 al 9 es: 7(6)(5) = 210.

Este ejercicio corresponde a ordenar objetos distintos pero se necesitan menos objetos de los que
se tiene disponibles, es decir, el problema consiste en saber cuántas configuraciones de longitud k
se pueden formar si se tiene n objetos disponibles (n > k). A estas configuraciones les llamaremos
arreglos (o variaciones) y al total de arreglos de longitud k teniendo n objetos disponibles lo
representaremos por Akn (En algunos libros se utiliza la notación n Pk , Pnk o Pkn en lugar de Akn ).
Ası́ en el ejemplo:

20
A37 = 7(6)(5)
4!
= 7(6)(5)
4!
7!
=
4!
7!
=
(7 − 3)!

A diferencia de las permutaciones (Pn = n!) donde se trata solo de ordenar los objetos disponi-
bles, en el caso de los arreglos, además de ordenar, se da también un proceso de selección porque
se debe decidir cuáles objetos de todos los disponibles serán los que se ordenen, por lo que otra
manera de obtener la solución del ejemplo 3 es seleccionando 3 dı́gitos de los 7 disponibles, para
luego ordenar los 3 dı́gitos seleccionados, por lo que se pueden formar C73 (3!) números de 3 dı́gitos
utilizando los dı́gitos del 3 al 9.

En general:

Akn = n(n − 1)(n − 2)...(n − k + 2)(n − k + 1)


(n − k)!
= n(n − 1)(n − 2)...(n − k + 2)(n − k + 1)
(n − k)!
n!
=
(n − k)!

y también: Akn = Cnk (k!).

3.3. Permutaciones con objetos indistinguibles


Ejemplo 4 ¿De cuántas maneras se pueden ordenar en fila 4 bolitas rojas, 2 azules y 3 verdes si
las bolitas del mismo color son idénticas entre sı́?

Si todas las bolitas fueran distintas entonces habrı́a 9! maneras de ordenarlas. Pero como hay
bolitas que son idénticas, entre las 9! maneras hay arreglos que se han contado más de una vez.
De hecho, si se tienen en una fila las 9 bolitas, y se intercambian entre sı́ las dos azules, el arre-
glo sigue siendo el mismo, de igual manera, si las 3 bolitas verdes se cambian de posición entre
sı́ entonces el arreglo sigue viéndose igual y lo mismo sucede con las rojas. Es decir que una vez
decidido en cuáles posiciones se ponen las bolitas azules, en cuáles las verdes y en cuáles las rojas,
hay 2!(3!)(4!) maneras de ubicar las bolitas en las posiciones correspondientes, de manera que los
arreglos resultantes son todos idénticos, lo que significa que entre las 9! maneras, cada arreglo se
ha contado 2!(3!)(4!) veces. Por lo tanto el número de maneras de ordenar en fila las 9 bolitas es
9!
.
2!(3!)(4!)

21
3.4. Permutaciones en arreglos circulares
Ejemplo 5 ¿De cuántas maneras pueden sentarse 5 niños(as) en un carrusel con 5 caballitos,
sı́ no hay manera de distinguir un caballito de otro?

Como los caballitos son idénticos y además el carrusel puede girar (no hay referencia), la única
manera de distinguir una forma de sentar a los niños(as) con respecto de otra es observando quién
queda a la par de quien, o siendo más precisos, observando a cada niño(a) y a quien está sentado a
su derecha. Una manera de encontrar la solución es sentando un niño(a) en cualquier caballito solo
para que sirva de referencia y ası́ los 4 caballitos restantes ya se pueden diferenciar entre sı́ según su
posicı́on respecto al que ya está sentado, ası́ estos 4 caballitos diferentes pueden ser ocupados de 4!
maneras. Otra forma es suponer que todos los 5 caballitos son distintos y que pueden ser ocupados
de 5! maneras, pero que hay 5 formas de rotar el carrusel manteniendo la posición relativa de los
niños(as), es decir que cada manera de sentarlos se ha contado 5 veces por lo que la respuesta es
5!
en realidad = 4!.
5

3.5. Ejemplos.
Ejemplo 6 ¿De cuántas maneras pueden sentarse 3 niños y 3 niñas en una fila de asientos, sı́ las
tres niñas desean sentarse en forma consecutiva?

Dado que las tres niñas deben sentarse juntas las tres, pueden ser consideradas como un bloque
el cual puede variar su posición junto con la de los tres niños, es decir que en principio se tiene
que contar las maneras de ordenar cuatro objetos: un bloque y tres niños, eso puede hacerse de 4!
maneras. Pero también, las tres niñas pueden variar su posición dentro del bloque de 3! maneras.
Por el principio de multiplicación, el total de maneras en que pueden sentarse los tres niños y las
tres niñas en la fila de asientos de forma que las tres niñas queden juntas es 4!(3!).

Ejemplo 7 ¿De cuántas maneras pueden sentarse 3 niños y 3 niñas en una fila de asientos, sı́ no
deben quedar las tres niñas juntas?

El total de maneras en que pueden sentarse las 6 personas es 6! y considerando la posición


de las tres niñas, todas estas maneras de sentarse pueden separarse en dos grupos: aquellas en
los que las niñas quedan las tres juntas y aquellas en los que no. Si se descuentan del total de
maneras aquellas en que las tres juntas, lo que quede es el resultado que se busca. Por lo tanto,
hay 6! − 4!(3!) maneras en que se pueden sentar los tres niños y las tres niñas de forma que no
queden las tres juntas.

Ejemplo 8 ¿De cuántas maneras pueden sentarse 3 niños y 3 niñas en una fila de asientos, sı́ no
deben quedar niñas consecutivas?

En este caso se desea que las niñas queden separadas, para ello, pueden ordenarse primero solo
a los niños, lo cual puede hacerse de 3! maneras. Luego, para que queden separadas, las niñas
pueden ubicarse en los espacios que quedan entre los niños o incluso en los extremos, por lo que se
tienen 4 espacios disponibles de los cuales se seleccionan 3, lo cual puede hacerse de C43 maneras y
finalmente se ordenan las niñas en esos tres lugares, lo que puede hacerse también de 3! maneras.
Ası́ que hay 3!(C43 )(3!) maneras de sentar a los niõs y a las niñas de forma que las niñas queden
separadas.

22
3.6. Ejercicios.
Indicaciones: Resuelva en forma clara y ordenada cada uno de los problemas que se le pre-
sentan, dejando constancia de sus soluciones.

1. Una profesora distribuye 5 caramelos distintos entre sus 8 estudiantes de tal forma que cada
estudiante recibe a lo sumo un caramelo, ¿De cuántas formas puede hacerlo? ¿Cuántas formas
serán si ya no se restringe la cantidad que puede recibir cada estudiante?

2. En una biblioteca hay 9 libros latinos y 6 griegos (todos los libros son distintos). ¿De cuántas
maneras pueden colocarse en un estante 5 de ellos, de los cuales 3 sean latinos y 2 griegos?

3. ¿Cuántas cadenas de 6 letras se pueden formar si todas sus letras son distintas y además
deben cumplir que:

a) las primeras dos son vocales y las últimas cuatro son consonantes.
b) las primeras dos son vocales.
c) tiene dos vocales y cuatro consonantes.
d ) tiene tres vocales y tres consonantes.
e) debe tener exactamente tres vocales que deben aparecer juntas.
f ) debe tener exactamente tres vocales que no deben aparecer las tres juntas.
g) debe tener exactamente tres vocales pero no pueden quedar ni siquiera dos de ellas
juntas.

4. Se desea formar un comité con un presidente, dos secretarios y tres tesoreros, para lo cuál
se dispone de 32 postulantes que pueden desempeñar cualquiera de los cargos. ¿De cuántas
maneras distintas puede quedar formado el comité?

5. Determine la cantidad de formas en que se pueden colocar en la primera fila del tablero de
ajedrez, las piezas: 2 torres, 2 caballos, 2 alfiles, el rey y la reina.

6. ¿Cuántas permutaciones se pueden hacer con las letras de la plabra MISSISSIPPI? ¿Cuántas
de éstas no contienen dos o más letras I consecutivas?

7. Cuatro bailarines y cuatro bailarinas interpretan una danza que consiste en formar una ronda
tomados de la mano. ¿De cuántas formas pueden ubicarse si en la figura deben aparecer
alternadamente hombres y mujeres?

8. En una mesa redonda hay 5 asientos y 7 personas que quieren sentarse (dos se quedarán de
pie). ¿De cuántas formas pueden hacerlo si la persona 1 es enemiga de la persona 2 y si se
sienta una no se sienta la otra?

9. Se tienen 12 personas y dos ruedas de asientos, cada una de seis lugares, una dentro de la
otra, tal como se muestra en la siguiente figura. ¿De cuántas formas distintas pueden ubicarse
las doce personas en los doce asientos?

23
10. ¿De cuántas maneras se podrán ubicar los números del 1 al 7 en la siguiente figura?

11. Cada una de las diagonales y lados de un polı́gono regular de n lados se colorean de azul o
de rojo. ¿De cuántas formas puede hacerse la coloración?
12. Mynor tiene 5 amigas. Desea invitarlas a cenar en grupos de tres por 5 dı́as consecutivos de
tal forma que cada dı́a invite a un grupo diferente. ¿De cuántas formas puede Mynor hacer
la selección?
13. Seis matrimonios se reúnen a cenar en una mesa circular. ¿De cuántas formas pueden ubicarse,
si cada hombre debe estar entre dos mujeres y los miembros de cada pareja deben estar juntos?
14. Para entrar a su casa, Manuel debe subir 7 escalones. ¿De cuántas maneras puede hacerlo si
nunca sube más de dos escalones a la vez?
n!
15. Demostrar que si n y p son enteros positivos y n es el doble que p, entonces p es entero.
2
16. ¿Cuántos números de dos mil catorce dı́gitos cumplen que el producto de todos sus dı́gitos
es par?
17. ¿De cuantas maneras pueden sentarse 4 parejas de casados al rededor de una mesa redonda?
¿Y si cada esposo quiere estar justo enfrente de su esposa?
18. Una madre tiene 2 manzanas y 3 peras, y le da a su hija una fruta cada dı́a (de lunes a
viernes), ¿De cuántas formas puede hacerlo, considerando que es irrelevante hacer distinción
entre una manzana y la otra o entre una pera y otra? ¿De cuántas formas podrá hacerlo si
tiene además 4 naranjas (siempre da una fruta cada dı́a de lunes a viernes)?

24
Capı́tulo 4
4. Teorema del Binomio
4.1. El triángulo de Pascal
Acomodemos en una tabla los valores de Cnk con los valores de n por filas y los de k por
columnas:

También pueden ubicarse en un arreglo simétrico de la manera siguiente:

La Identidad de Pascal nos dice que todo elemento del triángulo es igual a la suma de los dos
que se encuentran directamente sobre él. Esto quiere decir que si construimos un arreglo triangular
de números (comenzando y terminando cada fila con 1) de modo que todo número interior sea la
suma de los dos que están encima de él, los números que aparecen son precisamente los números
combinatorios. El arreglo que se forma se conoce como triángulo de Pascal.

El triángulo de Pascal encierra muchas relaciones numéricas, por ejemplo, la suma de todos los
números en la n-ésima fila es 2n . Sin embargo, quizás la relación más interesante en el triángulo
de Pascal tiene que ver con el Teorema del Binomio.

Consideremos (a + b)5 . Al desarrollarlo, ¿con qué coeficiente aparece el término cuya parte
literal es a3 b2 ?

Para desarrollar (a+b)5 = (a+b)(a+b)(a+b)(a+b)(a+b), de cada factor hay que seleccionar la


a o la b y luego multiplicar esos cinco términos. Por ejemplo el arreglo aabba representa el escoger
la a en los factores primero, segundo y quinto, y escoger la b en los factores tercero y cuarto. Como

25
en cada factor hay dos opciones para elegir, la cantidad de posibles arreglos es 25 = 32, pero de
éstos arreglos algunos serán términos semejantes, por ejemplo aabba, bbaaa, ababa y baaab son
algunos de los arreglos que corresponden al término a3 b2 , de manera que el coeficiente del término
con parte literal a3 b2 en el desarrollo de (a+b)5 se obtiene al contar la cantidad de arreglos de cinco
letras que pueden hacerse con tres a y dos b, esto es C53 porque solo se necesita seleccionar entre los
5 espacios disponibles los 3 que serán ocupados por las a y luego las b se colocan automáticamente,
esto equivale a decir que de los cinco factores se puede seleccionar los tres en los que se tomará la
a , lo cual puede hacerse de C53 maneras y en los dos factores restantes se seleccionará la b (lo cual
no agrega algo extra al resultado).

Ahora si estudiamos el caso general, al desarrollar (x + y)n , como son n factores cada arreglo
estará formado por n letras de las cuales algunas, digamos k (con 0 ≤ k ≤ n), serán y y el resto
n − k serán x, por lo que el término con parte literal xn−k y k tendrá coeficiente Cnk . Además, como
hay n + 1 opciones para los valores de k (es decir para la cantidad de y’s en el arreglo), el desarrollo
de (x + y)n tendrá n + 1 términos. Se obtiene la siguiente identidad algebraica:

4.2. Teorema (Binomio de Newton):


El desarrollo del binomio (x + y)n es:
n
X
(x + y)n = Cn0 xn + Cn1 xn−1 y + Cn2 xn−2 y 2 + ... + Cnn−1 xy n−1 + Cnn y n = Cnk xn−k y k .
k=0

Observación 1: Si se consideran los términos en el orden inverso y utilizando la propiedad


simétrica de los números combinatorios, podrı́a escribirse la identidad de la manera siguiente:
n
X
n
(x + y) = Cnk xk y n−k .
k=0

Observación 2:Una identidad ya conocida se obtiene fácilmente sustituyendo en la igualdad


anterior los valores x = 1, y = 1 , ¿cuál es esa identidad?

4.3. Ejercicios.
Indicaciones: Resuelva en forma clara y ordenada cada uno de los problemas que se le pre-
sentan, dejando constancia de sus soluciones

1. Obtener el resultado de desarrollar (x + 5y 3 )4 .


2. Si se obtiene la expansión de (1 + x)500 encontrar el coeficiente de x276 .
3. Encontrar el término que no contiene a x en el desarrollo de
9


1
x+ √ .
4
x

26
4. Justifique utilizando el triángulo de Pascal las identidades combinatorias de Chu Shih Chieh
r
Crr + Cr+1 r+1
+ · · · + Cnr = Cn+1
Cr0 + Cr+1
1 k
+ · · · + Cr+k k
= Cr+k+1 .
5. Utilizar el teorema del binomio para probar la fórmula
           
n n n n k n n n
− + − + ... + (−1) + ... + (−1) = 0.
0 1 2 3 k n
Observa que la identidad anterior equivale a decir que en una fila del triángulo de Pascal, la
suma de los combinatorios pares es igual a la suma de los impares.
6. Demuestre n
X
3r Cnr = 4n
r=0

7. Encuentra el coeficiente de xn en la expansión de


(1 + x)2n + x(1 + x)2n−1 + x2 (1 + x)2n−2 + ... + xn (1 + x)n .

8. Probar que para cualquier número natural se tiene la fórmula


 2  2  2  2  
n n n n 2n
+ + + ... + = ,
0 1 2 n n
utilizando el teorema del binomio. (Sugerencia: Estudia algún coeficiente de (1 + x)2n )
9. Escribe en cada una de las casillas vacı́as de la pirámide siguiente un número natural mayor
que 1, de modo que: el número escrito en cada casilla sea igual al producto de los números
escritos en las dos casillas sobre las que está apoyada.

10. Considere los reales positivos p y q tales que p + q = 1. Demuestre que:


  n
n k (n−k) X
rk = p q , 0 ≤ k ≤ n =⇒ (k ∗ rk ) = np
k k=0

11. Se construye un triángulo aritmético de la siguiente manera: primero se escriben en fila los
números del 0 al 500, luego, cada número de la segunda fila es el resultado de sumar los dos
números consecutivos de la primera fila que quedan justo sobre él y ası́ se continúa formando
el triángulo hasta que una de las filas está formada por solo un número. Demostrar que ese
último número es múltiplo de 500.

27
Capı́tulo 5
5. Inclusión-exclusión y Desordenes
5.1. Inclusión-exclusión
Problema 1. ¿Cuántos enteros entre 1 y 6300 inclusive, no son divisibles entre 5?

Solución: Para resolver este problema primero encontremos los números que son divisibles por
5. Puesto que cada 5 números consecutivos exactamente uno de ellos es múltiplo de 5, es decir, de
los 6300 números, hay exactamente 6300
5
, es decir 1260, son divisibles por 5. Por lo que la respuesta
es
6300 − 1260 = 5040
Problema 2. ¿Cuántos enteros entre 1 y 6300 inclusive, no son divisibles entre 5 ni entre 3?

Para resolver este problema podemos hacerlo de igual forma como se desarrollo el problema 1,
y decir que el número de enteros que es divisible por 5 es 1260 y que el número de divisores de 3
es 6300
3
, osea 2100. Pero
6300 − 2100 − 1260 = 2940
no es la solución ya que en este problema los números 15, 30, 45, . . . son divisibles tanto por 3
como por 5 que habrán sido eliminados dos veces de los 6300 enteros, por lo que debemos sumar
el número de enteros divisibles tanto por 3 como por 5, es decir los divisible por 15 los cuales son
6300
15
, es decir, 420 números por lo que la respuesta es

6300 − 1260 − 2100 + 420 = 3360

Definición:
Para calcular la cardinalidad de A1 ∪ A2 ∪ · · · ∪ An se debe calcular la cardinalidad de todas las
posibles intersecciones de conjuntos A1 , A2 , ..., An sumar los resultados obtenidos al intersectar
un número impar de conjuntos y restar los resultados obtenidos al intersectar un número par de
conjuntos.

Los términos “inclusión-exclusión” indican que hay que incluir o sumar las cardinalidades de
los conjuntos, después excluir o restar las cardinalidades de las intersecciones de dos conjuntos,
luego incluir o sumar las cardinalidades de todas las intersecciones de tres conjuntos, etc, es decir:
n
X
|A1 ∪ A2 ∪ · · · ∪ An | = (−1)j+1 αj (1)
j=1

En donde los αj son las sumas de las cardinalidades de todas las posiles intersecciones de j con-
juntos, de la manera siguiente:

28
α1 = |A1 | + |A2 | + · · · + |An |
α2 = |A1 ∩ A2 | + |A1 ∩ A3 | + · · · + |An−1 ∩ An |
α3 = |A1 ∩ A2 ∩ A3 | + |A1 ∩ A2 ∩ A4 | + · · · + |An−2 ∩ An−1 ∩ An |
..
.
αn = |A1 ∩ A2 ∩ A3 ∩ · · · ∩ An |

Prueba: Sea x ∈ A1 ∪ A2 ∪ · · · ∪ An , luego se perseguirá que la formula (1) cuente solamente


una vez cada elemento en A1 ∪ A2 ∪ · · · ∪ An , ahora asumiendo que x ∈ A1 , A2 , . . . , Ap pero
x∈/ Ap+1 , Ap+2 , . . . , An , entonces x será contado en cada αi de la siguiente manera:

α1 = p
 
p
α2 =
2
 
p
α3 =
3
..
.
 
p
αp =
p

Luegoaplicando (1):
p − p2 + p3 − p4 + · · · + (−1)p−1 pp
  

Lo cual equivale a:

p   X p  
X
j+1 p j−1 p
(−1) = (−1)
j=1
j j=1
j
p  
j−1 p
X
= (−1) +1
j=0
j
=0+1
=1

Con lo cual se puede concluir que el elemento x será contado una sola vez utilizando la fórmula
(1).

5.1.1. Ejercicios.
Indicaciones: Resuelva clara y ordenadamente cada uno de los problemas que se le presentan,
dejando constancia de sus soluciones

29
1. ¿Cuántos números entre 1 y 6300 inclusive son divisibles entre 3, entre 5 y entre 7?

2. De los números entre 1 y 200, ¿Cuántos no son divisibles entre 4, ni entre 6, ni entre 9?

3. ¿Cuántos números del 1 al 1000000 no son ni cuadrados perfectos, ni cubos perfectos ni


potencias cuartas perfectas?

4. Sea S el conjunto de números de 3 dı́gitos abc tal que a, b, c ∈ {1, 2, · · · , 9} y a, b, c son


distintos. (Por lo tanto, 489 ∈ S, pero 313 ∈
/ S y 507 ∈
/ S.) Encuentra el número de elementos
abc en S tal que a 6= 3, b 6= 5 y c 6= 7.

5. En un grupo de 30 personas, 10 hablan inglés, 12 hablan castellano y 10 hablan francés. Se


sabe que 5 hablan inglés y castellano, 5 castellano y francés, y 7 inglés y francés. Tres personas
hablan los tres idiomas. ¿Cuántas personas no hablan ninguno de estos tres idiomas?

6. En un grupo de 100 hindúes hay 40 que hablan hindi, 40 que hablan bangalı́ y 20 que hablan
penjabi. Hay 20 que hablan hindi y benbalı́ y 5 que hablan hindi y penjabi. Hay 31 que
hablan al menos dos de estas tres lenguas y 33 que no hablan ninguna de ellas. ¿Cuántos
hablan las tres lenguas?

7. Cuatrocientos niños forman un cı́rculo y los númeramos 1, 2,..., 400. Sea k, 1 ≤ k ≤ 400
un entero fijo. Marcamos cada k niño deteniéndonos cuando marcamos un niño por segunda
vez. Por ejemplo, si k = 6, comenzamos marcando 6, 12, 18, ..., 396. Luego nos toca marcar
al niño 2. Seguimos marcando a los niños 8, 14, 20,...,398. Nos toca marcar a los niños 4, 10,
16, . . ., 400. El proximo niño a marcar es el sexto, que lo marcamos pues por segunda vez.
Notamos que dejamos sin marcar a los niños 1, 3, 5, 7, 9, 11, ...,399 esto es, los enteros de
la forma 6k ± 1, 6k ± 3 entre 1 y 400. ¿Para cuántos valores de k serán marcados todos los
niños al menos una vez?

5.2. Desórdenes

Ejemplo: Si consideramos el orden de los números naturales 1, 2, 3, 4, en la permutación 3142


ningún elemento está en su posición natural. A una permutación con tal propiedad le denomina-
remos un desorden o un desarreglo. ¿De los 24 posibles ordenamientos de tales números, cuántos
desórdenes existen?

Denotemos por D4 tal número de desórdenes; por F1 el número de las permutaciones que dejan
fijo un elemento en su posición; por F2 las que dejan en su posición dos de los 4 elementos, por F3
las que dejan en su puesto tres de los 4 elementos; de manera completamente similar definiremos
F4 . Ası́, por el principio de inclusión-exclusión tenemos el siguiente resultado:

D4 = 4! − F1 + F2 − F3 + F4
Ahora bien, F1 se descompone en los que dejan fijo el 1 en su posición, que son 6, los que dejan
fijo el 2, que son otras 6; hay 6 que dejan el 3 en su posición y 6 que dejan fijo el 4; ası́, F1 es 24.
De las que dejan fijos dos de los cuatro elementos están los que dejan fijos el 1 y 2, los que dejan
fijos el 1 y 3, el 1 y 4; los que dejan fijos el 2 y 3, el 2 y 4; finalmente tenemos los que dejan fijos el
3 y el 4. Como en cada uno de estos caso, que son seis, el número de permutaciones es 2, resulta
que F2 es 12. El caso de que dejen fijos tres elementos contiene los casos siguientes: que dejen fijos

30
1, 2, 3; 1, 2, 4; 1, 3, 4 y el caso que dejen fijos 2, 3, 4; en total son cuatro casos y cada uno de ellos
tiene una permutación por lo que F3 es igual a 4. Finalmente, F4 contiene una única permutación,
es decir F4 es 1. En resumen tenemos:

D4 = 24 − 24 + 12 − 4 + 1 = 9
Hay en consecuencia 9 desórdenes en las permutaciones de orden 4.

Esta claro que esta relación se puede generalizar.

Teorema 1 El total de desórdenes de orden n, denotado por Dn , es


n  
k n
X
Dn = (−1) (n − k)!
k=0
k

Demostración El total de permutaciones es Pn = n!. Tomando la misma notación del ejemplo


anterior, Fk denota aquellas permutaciones que tienen a k (al menos) de sus elementos en la
posición que les corresponde; ası́, el total de permutaciones de Fk lo contamos primero escogiendo
los k que quedarán en la posición que les corresponde, lo cuál es posible hacerlo de Cnk formas, y
luego permutando los restantes n−k objetos, lo cual es posible hacerlo de Pn−k = (n−k)! formas, y
por el principio de la multiplicación, Fk = Cnk (n−k)!. Luego, por el principio de inclusión-exclusión:

Dn = n! − F1 + F2 − F3 + · · · + (−1)k Fk + · · · + (−1)n Fn
= (−1)0 Cn0 (n − 0)! + (−1)1 Cn1 (n − 1)! + (−1)2 Cn2 (n − 2)! + · · · + (−1)n Cnn (n − n)!
n  
k n
X
= (−1) (n − k)!
k=0
k

Observe que esa expresión se puede manipular algebraicamente, y reescribirse como


n
X (−1)k
Dn = n!
k=0
k!

5.2.1. Ejercicios.
Indicaciones: Resuelva en forma clara y ordenada cada uno de los problemas que se le pre-
sentan, dejando constancia de sus soluciones.

1. Calcular D3 y D5 .

2. Hallar el número de permutaciones de los enteros del 1 al 10 inclusive tal que ningún número
esta en su lugar habitual y si en los primeros cinco lugares están:

a) 6, 7, 8, 9, 10; en algún orden.


b) 1, 2, 3, 4, 5; en algún orden.

3. Hallar el número de permutaciones de 1, 2, 3, 4, 5, 6, 7 que no tenga a 1 en el primer lugar


a 4 en el cuarto ni a 7 en el séptimo lugar.

31
4. ¿Cuántas permutaciones de los enteros del 1 al 9 inclusive tienen exactamente tres de sus
números en sus posiciones naturales y los otros seis no?

5. Se tiene un tablero de 4 × 4 y cuatro colores diferentes se pinta cada cuadrito de la primera


fila de un color diferente de tal manera que cuando se pinta la segunda fila cada cuadrito es
de diferente color al cuadrito de la parte superior, de forma análoga se colorean las demás
filas. ¿De cuántas maneras se puede colorear el tablero?

6. En cada uno de los vértices de un cubo hay una mosca. Al sonar un silbato, cada una de las
moscas vuela a alguno de los vértices del cubo situado en una misma cara que el vértice de
donde partió, pero diagonalmente opuesto a éste. Al sonar el silbato, ¿de cuántas maneras
pueden volar las moscas de modo que en ningún vértice queden dos o más moscas?

32
Capı́tulo 6
6. Separadores
6.1. Ejemplos.
Considere el siguiente problema:

Ejemplo 9 Ana quiere comprar 10 dulces para regalárselos a sus primitos; en la tienda hay dulces
de tres sabores, menta, fresa y limón, ¿De cuántas formas puede escogerlos?

Si llamamos m a la cantidad de dulces de menta, f la cantidad de dulces de fresa y l la cantidad


de dulces de limón, debe cumplirse que m + f + l = 10, y obviamente cada uno de estos números es
mayor o igual a cero. Como siempre, lo mejor es analizar algunos casos particulares, por ejemplo
m = 0, entonces f + l = 10, y las parejas solución son

(f, l) = (0, 10), (1, 9), (2, 8), (3, 7), (4, 6), (5, 5), (6, 4), (7, 3), (8, 2), (9, 1), (10, 0)

en total hay 11 posibilidades. Si analizamos el resto de casos (todos disjuntos) m = 1, 2, . . . , 10,


se obtendrán respectivamente 10, 9, . . . , 1 posibilidades, por lo que la cantidad de formas que Ana
puede hacer la compra es 11 + 10 + · · · + 1 = 66.

Este método resuelve el problema y las cuentas no son largas ni difı́ciles, sin embargo no nos
da idea de como abordar un problema con más variables o con números más grandes. Por ejemplo,
si los sabores disponibles fueran 8 y la cantidad de dulces que Ana compra es 100, el problema se
vuelve muchı́simo más difı́cil y el método anterior no funcionará bien.

Si se observa, este problema es distinto a todos los estudiados hasta el momento, y básicamente
se trata de buscar combinaciones de objetos, pero no todos los objetos son distintos (para el ca-
so, los dulces de fresa los consideramos todos iguales, los de menta también y los de limón también).

Hay una forma muy ingeniosa de resolver este problema. Haremos lo siguiente: los 10 dulces los
vamos a interpretar como 10 objetos iguales, 10 bolas por ejemplo, y para distinguir cuáles son de
cada sabor, incluiremos 2 “separadores”; luego, estos 12 objetos se permutan, m es la cantidad de
bolas que quedan a la izquierda del primer separador, f es la cantidad de bolas que quedan entre
los separadores y l es la cantidad de bolas que quedan a la derecha del segundo separador. Además,
12!
la cantidad de permutaciones con repetición con 10 bolas y 2 separadores es P (10, 2) = = 66.
10!2!




|{z } | {z } | {z }
m f l

El problema general se resuelve de la misma forma:

33
Teorema 2 Dada una colección de objetos clasificados en k tipos de objetos (los objetos del mismo
tipo son iguales entre sı́, y distintos de cualquier objeto de otro tipo), el total de formas de escoger
n objetos es
(n + k − 1)!
P (n, k − 1) =
n!(k − 1)!

| | || | · · · |
Demostración: Se considera en principio que los n objetos son todos iguales, y para distribuir
los objetos entre las posibles k clases, se agregan k − 1 separadores. El total de configuraciones es
igual a las permutaciones con repetición P (n, k − 1).
Observe que la respuesta puede verse como un combinatorio también: en total, se tienen n+k−1
espacios y se escogen los n (o bien los k − 1) en los que se ubican las bolas (o bien los separadores),
por lo que la cantidad de configuraciones buscadas es

n k−1
Cn+k−1 o bien Cn+k−1
Finalmente, una versión muy utilizada de separadores es la siguiente:

Teorema 3 El total de soluciones enteras no negativas de la ecuación x1 + x2 + · · · + xk = n es


n
Cn+k−1

La relación con el problema anterior es evidente, porque xi representa la cantidad de objetos


del tipo i; el particular, el problema de Ana es equivalente a resolver la ecuación x1 + x2 + x3 = 10,
con xi ∈ Z+ 0 . Note que hay configuraciones que tienen cero bolas de algún tipo, esto, en el esquema
de los separadores, se da cuando los separadores están juntos, o cuando un separador está a la
izquierda de todas las bolas o a la derecha de todas las bolas.

Otro detalle importante que comentar es que a veces se busca configuraciones que tengan al
menos uno de cada tipo, es decir, xi ≥ 1. En tal caso los separadores se ubican únicamente en los
n − 1 espacios entre las n bolas, a lo sumo un separador por espacio; ası́, el total de configuraciones
k−1
con esta nueva restricción es Cn−1 .

6.2. Ejercicios.
Indicaciones: Resuelva en forma clara y ordenada cada uno de los problemas que se le pre-
sentan, dejando constancia de sus soluciones.

1. Se han encargado 20 pupusas de entre los siguientes tipos: revueltas, de queso, de chicharrón,
de frijol con queso, de queso con loroco y de ayote.

a) ¿De cuántas formas puede hacerse la compra?


b) ¿De cuántas formas si se tiene que llevar al menos 7 de queso?.
c) ¿De cuántas formas si se tiene que llevar a lo sumo 2 de chicharrón y 10 de ayote?.
d ) ¿De cuántas formas si se tiene que llevar al menos 3 de cada clase?.

34
2. Se tienen seis cajas numeradas del 1 al 6. De cuántas formas se pueden repartir 20 pelotas
entre las cajas de manera que ninguna quede vacı́a.

3. Cuántas soluciones tiene la ecuación x1 + x2 + x3 + x4 + x5 = 44

a) en enteros no negativos
b) en enteros positivos.
c) en enteros no negativos y con x2 ≥ 5.
d) en enteros positivos y con x2 ≤ 5.
e) en enteros positivos, con x2 ≤ 5 y con x3 < 8.

4. ¿De cuántas formas pueden distribuirse 20 bolas iguales en 6 cajas, de tal forma que en la
primera caja hay al menos 4 bolas y en la última caja no más de 5? ¿Y si también en la
penúltima no pueden haber más de 5?

5. ¿De cuántas maneras se puede seleccionar 10 dı́gitos 0, 1 ó 2 (no importa el orden)?

6. ¿Cuántas cadenas existen de 10 dı́gitos ternarios (0, 1 ó 2) que contengan exactamente dos
0, tres 1, y cinco 2?

7. ¿Cuántas maneras hay de distribuir 4 pelotas negras, 4 blancas y 4 azules en 6 cajas distintas?

8. ¿De cuántas maneras se pueden acomodar en hilera 5 pelotas rojas, 5 azules y 5 verdes de
tal manera que no queden 2 pelotas azules juntas? (Encontrar la solución de dos maneras)

9. ¿De cuántas formas pueden ordenarse n ceros y k − 1 unos si no hay dos 1 consecutivos?

10. Determine el número de formas en que pueden ordenarse en un estante 4 libros distintos
de Combinatoria, 5 libros distintos de Geometrı́a, 3 libros distintos de Álgebra y 8 libros
distintos de Cálculo, si los de Geometrı́a deben estar siempre antes que los de álgebra.

11. Existen 5 formas de expresar el número 4 como suma de dos enteros no negativos tomando
en cuenta el orden: 4 = 0 + 4 = 1 + 3 = 2 + 2 = 3 + 1 = 4 + 0. Dados los naturales r y n,
determine:

a) El número de formas de expresar 200 en r sumandos.


b) El número de formas de expresar n en 200 sumandos.
c) El número de formas de expresar n en r sumandos tales que todos sean mayores o
iguales que 5.

12. ¿Cuántas soluciones existen para la desigualdad

x1 + x2 + x3 ≤ 11

donde x1 , x2 y x3 son enteros no negativos?

13. En la expansión de (a + b + c + d)48

a) Encontrar el coeficiente de a8 b10 c15 d15 .

35
b) Determinar el número de términos de la expansión (cuando ya se han reducido términos
semejantes).
c) ¿Cuál es el total que se obtiene al sumar todos los coeficientes de esos términos?

14. Determine el número de términos no semejantes de la expansión de (x1 + x2 + x3 + · · · + xn−1 )n−1 .

15. ¿Cuántas soluciones hay, entre 1 y 9 inclusive, de la ecuación x1 + x2 + x3 + x4 = 26?

16. Si Lorena tira un dado cinco veces, ¿cuál es la probabilidad de que la suma de sus cinco
tiradas sea 20?

17. Se tiran 12 dados idénticos al aire, ¿cuál es la probabilidad de que exactamente uno de los
números 1, 2, . . . , 6 no aparezca?

18. Determine el número de soluciones enteras no negativas de 3x1 + 5x2 + x3 + x4 + x5 = 20.

19. Determine el número de soluciones enteras no negativas de rx1 + x2 + x3 + · · · + xn = kr.


6
20. Determine el coeficiente de x5 en la expansión de (1 + x + x2 + · · · + x1000 ) .

36
Capı́tulo 7
7. Comparaciones
7.1. Ejercicios.
Indicaciones: Resuelva en forma clara y ordenada cada uno de los problemas que se le pre-
sentan, dejando constancia de sus soluciones.
k−1 k−2
k
1. Demuestre por caminos y por conjuntos la igualdad: Cnk = C20 Cn−2 + C21 Cn−2 + C22 Cn−2 .
2. Demuestre por conjuntos la identidad
   
n−2 n
n(n − 1) = k(k − 1) .
k−2 k
3. Demuestre por diversos métodos (caminos, conjuntos y cadenas binarias) la identidad de
Vandermonde
0
Cm Cnr + Cm
1
Cnr−1 + · · · + Cm
r
Cn0 = Cm+n
r

4. Demuestre por caminos y por conjuntos la identidad combinatoria de Chu Shih Chieh
Crr + Cr+1
r
+ · · · + Cnr = Cn+1
r+1

5. Probar la formula de Gauss para sumar los primeros n números enteros positivos,
n(n + 1)
1 + 2 + 3 + 4 + ··· + n =
2
con un argumento a partir de la figura siguiente:

6. Demuestra que si n ≥ k ≥ r ≥ s, entoces


       
n k r n n−s n−r
= .
k r s s r−s k−r
(Sugerencia: Imagina que de los n puntos pintas k de un color, luego de los k puntos pintas
r de otro color y finalmente de los r pintas s de otro color)
7. Demuestre la identidad
q−1 1 q q
Cpq Cr0 + Cp−1 0
Cr+1 + · · · + Cp−q Cr+q = Cp+r+1
8. Demuestra que si m, n son enteros positivos entonces
n   
X
k m+1 m+n−k
(−1) =0
k=0
k n − k

37
Capı́tulo 8
8. Principio de Casillas
Principio de las casillas (también llamado principio de los palomares)
Se dispone de n casillas para colocar m objetos y m > n, entonces en alguna casilla se deberán
colocarse por lo menos dos objetos.

Ejemplo 10 Un costal esta lleno de canicas de 20 colores distintos. Al azar se van sacando cani-
cas del costal. ¿Cuál es el mı́nimo de canicas que deben sacarse para garantizar que en la colección
tomada habra al menos 100 canicas del mismo color?

Solución
Primero notemos que si sacamos 20 canicas podrı́a ser que todas fueran de colores distintos, ası́ que
podemos garantizar que hay dos canicas del mismo color al sacáramos 21 canicas (aquı́ aplicamos el
principio de casillas ) de igual forma necesitaremos 41 (20 × 2 + 1) canicas para garantizar que hay
al menos 3 del mismo color, pues con 40 canicas puede ocurrir que cada color aparece exactamente
2 veces. Por lo que se necesitan 20 × 99 + 1 = 1981 canicas para que al menos se tengan 100 del
mismo color.

Ejemplo 11 Suponiendo que se tienen 27 números impares positivos menores que 100. Demostrar
que hay al menos dos de ellos cuya suma es 102.

Solución
Se puede intuir que lo que es conviente tomar son los 27 números como nuestros objetos y crear
26 casilleros que contengan a todos los impares menores que 100 con la propiedad de que si dos
números están en el mismo casillero, entonces su suma es 102.
Sabiendo eso, entonces es relativamente simple pensar que el número 3 debe estar en el mismo
casillero que el número 99, y asi mismo el 5 y el 97, el 7 y el 95, etc. Pero, ¿Qué sucede con el
número 1?. Es claro que no existe un impar menor que 100 que sumado con 1 dé 102. En ese
caso, lo que más conviene es dejarlo en un casillero aparte. De esa manera será imposible que dos
números distintos queden en ese casillero pues ese casillero es especial para el número 1. Lo mismo
se hace para el número 51. Sabiendo esto, podemos dividir el conjunto de los números impares
positivos menores que 100 en los siguientes casilleros:

De esta manera se ha dividido el conjunto en exactamente 26 casilleros distintos, y como en el


problema original se tienen 27 números impares positivos menores que 100, el Principio de Palomar
nos asegura que dos de ellos caerán en el mismo casillero. Y por la manera como se han construı́do
las casillas, la suma de ambos números será 102.
Al ver la resolución del problema anterior podrı́a surgir una duda: ¿Es 27 la cantidad mı́nima
de números impares que se debe tomar para asegurarse que haya un par que sume 102?. ¿Si se
toma sólo 26 se puede asegurar que hay un par que suma 102? La respuesta a la última pregunta es
negativa, de hecho, los casilleros que se construyerón facilitan la respuesta: basta tomar un número
de cada uno de los casilleros (que son 26) y se obtendrá un conjunto de 26 números de los cuales
ningún par de ellos suma 102.

38
Ejemplo 12 Sea un cuadrado de diagonal 3 en el que se marcan al azar 10 puntos. Demostrar
que siempre se pueden encontrar al menos 2 puntos que estén a una distancia no mayor a 1.

Solución
Se sabe que se van a marcar 10 puntos al interior del cuadrado; luego, si se crean 9 “casilleros” que
cubran completamente el cuadrado de modo que cada punto deba forzosamente entrar en alguno
de los casilleros, el principio del palomar asegurarı́a que al menos 2 puntos caerı́an en el mismo
casillero. Luego, para resolver el problema, bastarı́a encontrar una forma de crear 9 casilleros con
la gracia que si dos puntos caen en el mismo casillero entonces la distancia entre ambos sea menor
o igual que 1. Una forma de lograr esto es dividir el cuadrado en 9 cuadrados más pequeños de la
siguiente manera:

Y suponiendo que los bordes de los cuadrados se asignan arbitrariamente a alguno de los cua-
drados que bordea (por ejemplo, se puede decir que las lı́neas horizontales pertenecen al cuadrado
inmediatamente superior, las verticales pertenecen al cuadrado inmediatamente a la izquierda y los
vértices al cuadrado inmediatamente arriba y a la izquierda). De esta manera, cualquier punto que
se coloque en el cuadrado grande pertenecerá a uno y sólo uno de los cuadrados pequeños. Además
como el cuadrado grande tiene diagonal 3, los cuadrados pequeños tendrán diagonal 1. Luego, la
distancia máxima que puede haber entre dos puntos dentro de un mismo cuadrito será de 1 (la
diagonal del cuadrito). Ası́, el Principio del Palomar asegura que, al marcar 10 puntos, dos de
ellos deberán quedar en el mismo cuadrito, y por lo tanto la distancia entre dichos dos puntos
será menor o igual que 1.

Principio de las casillas infinito


Dado un conjunto infinito de objetos, si se reparten en un número finito de casillas, debe haber al
menos una casilla con una cantidad infinita de objetos.

8.1. Ejercicios.
1. ¿Cuántas personas se necesitan como mı́nimo para estar seguros que hay al menos dos que
cumplen años el mismo dı́a de la semana? ¿y para que hayan al menos 3? ¿y para que hayan
al menos n?
2. En un cajón hay calcetines negros, rojos, azules y blancos. ¿Cuál es el menor número de
calcetines que hay que sacar para estar seguros de que hay al menos dos del mismo color?
3. ¿Cuántas veces, como mı́nimo, debe lanzarse un par de dados para asegurarse que el puntaje
obtenido (la suma de los dados) se repita?
4. En una caja hay 10 libros en francés, 20 en castellano, 8 en alemán, 15 en ruso y 25 en
italiano. ¿Cuantos debo sacar para estar seguro de que tengo 12 en un mismo idioma?
5. A un estadio de fútbol han asistido 37000 espectadores. ¿Cuántos de ellos, como máximo,
puede asegurarse que cumplen años el mismo dı́a del año?

39
6. Una prueba de concurso posee diez preguntas de selección múltiple, con cinco alternativas
cada una. ¿De cuántas maneras diferentes se puede responder el examen? ¿Cuál es el número
mı́nimo de candidatos que deberán hacer el examen para garantizar que por lo menos dos de
ellos tendrán las mismas respuestas para todas las preguntas?

7. Un examen de admisión a la universidad tiene 100 preguntas de opción múltiple con 4


respuestas alternativas para cada pregunta. Si los alumnos (as) responden todas las preguntas
sin excepción:

a) ¿Cuántos alumnos(as) se necesitan para garantizar que hay dos de ellos con las mismas
respuestas en todo el examen?
b) ¿Cuántos alumnos(as) se necesitan para garantizar que hay 15 de ellos con las mismas
respuestas en todo el examen?

8. En un restaurante hay 95 mesas y un total de 465 sillas. ¿Podemos asegurar que hay una
mesa con 6 sillas?

9. Un grupo de 30 alumnos(as) hicieron un examen de Combinatoria. Si se sabe que entre


cualesquiera 10 de ellos, siempre hay dos que obtuvieron la misma nota, ¿Cuál es el máximo
de calificaciones diferentes que pudieron haber en el examen?

10. ¿Cuál es el máximo de cajas que pueden estar disponibles para colocar 100 bolitas, de manera
que con seguridad queden al menos 5 bolitas en alguna caja?

11. En una reunión hay n personas. Muestre que existen dos personas que conocen exactamente
al mismo número de otros participantes (admitimos que conocer es una relación simétrica,
es decir que, si a conoce a b, entonces b conoce a a).

12. Demuestre que si del subconjunto de números naturales 1, 2, . . . , 10 extraemos seis números,
con seguridad habrá dos que suman 11.

13. Se tienen los números 1, 2, . . . , 2n escritos en una pizarra. Se tachan n − 1 de ellos. Probar
que entre los números que quedaron sin tachar en la pizarra, hay al menos dos de ellos que
son consecutivos.

14. Hay 100 personas sentadas en una mesa circular a distancia constante entre sı́ y al menos 51 de
ellas son mujeres. Verificar que hay al menos 2 mujeres sentadas en posiciones diametralmente
opuestas.

15. Se tiene un conjunto de diez números naturales. Demostrar que hay al menos un par cuya
diferencia es múltiplo de 9.

16. Demostrar que un conjunto infinito de números naturales siempre tiene un subconjunto
infinito tal que cualesquiera dos elementos del subconjunto al restarlos se obtiene un múltiplo
de 2014

17. Considere los primos 2, 3, 5. El conjunto A está formado por los números naturales que
se generan multiplicando distintas potencias de estos primos, es decir, los números de la
forma 2α 3β 5γ con α, β, γ enteros no negativos. Demuestre que de cualquier escogitación de 9
números de A siempre hay 2 que al multiplicarlos generan un cuadrado perfecto.

40
18. De cinco puntos dentro o sobre los lados de un triángulo equilátero de lado 2 hay dos cuya
distancia entre ellos es menor o igual a 1.

19. ¿Cuál es el mayor número de reyes que pueden ser colocados en un tablero de ajedrez de
manera que ninguno dé jaque a ningún otro?

20. Se ubican puntos en el plano, cada uno de color rojo , azul, verde o negro ¿Cuántos puntos
deben ubicarse para estar seguros de que al menos 4 de esos puntos son rojos, o al menos 3
son azules, o al menos 5 son verdes o al menos 6 son negros?

21. Demuestre que si con 2 colores se pintan las diagonales de un pentágono regular, siempre
hay un vértice del que salen dos diagonales del mismo color.

22. Para una reunión cientı́fica se han contratado 5 traductores, que deberán cubrir 6 lenguas
diferentes. Si cada una de éstas requiere el empleo de 3 traductores, demostrar que alguno
de los intérpretes deberá traducir por lo menos 4 idiomas.

23. Demuestre que un triángulo equilátero de lado 1 no puede ser cubierto totalmente con dos
triángulos equiláteros de lados menores que 1.

24. Demuestre que dados 5 puntos en el plano con coordenadas enteras siempre hay dos tales
que su punto medio tiene coordenadas enteras.

25. Demuestre que dados 13 puntos en el plano con coordenadas enteras siempre hay 4 tales
que su baricentro tiene coordenadas enteras. (El baricentro de cuatro puntos es aquel cuyas
coordenadas son el promedio de las coordenadas correspondientes de los puntos).

26. Comprobar que en una reunión de 6 personas siempre pasa que 3 de ellas se conocen entre
sı́ o bien 3 de ellas no se conocen entre sı́.

27. Mostrar que cualesquiera 5 puntos colocados en un triángulo equilatero de lado 1 siempre
1
existen dos que se encuentran a una distancia es a lo sumo .
2
28. Mostrar que dado cualquier conjunto de 5 números siempre existen 3 cuya suma es divisible
por 3.

29. 17 personas se comunican por correo, enviando cada persona una carta a cada una de las
demás. En las cartas solo son discutidas tres temáticas distintas. Las cartas enviadas mutua-
mente entre dos personas tratan ambas a cerca de una sola de esas temáticas, dado que una
persona envı́a una carta y la persona a la que va dirigida le responde. Pruebe que hay un
grupo de al menos 3 personas tales que en todas las cartas que se enviaron entre sı́ discutieron
a cerca de la misma temática.

30. Los números naturales del 1 al 64 se colocan en las casillas de un tablero de ajedrez. Demostrar
que hay dos casillas abyacentes tales que los números escritos en ellas difieren en al menos 5.

31. Se tienen 1985 enteros positivos tales que ninguno tiene un divisor primo mayor a 23. De-
muestre que hay cuatro de ellos cuyo producto es la cuarta potencia de un entero.

41
32. En una retı́cula de n×5 puntos (es decir, un arreglo de puntos formando n filas y 5 columnas),
cada punto se colorea con uno de 4 colores. ¿Cuál es el mı́nimo valor de n para el cual se
puede asegurar que existe un rectángulo con vértices en la retı́cula cuyos cuatro vértices son
todos del mismo color?

42
Capı́tulo 9
9. Coloración y Paridad
9.1. Coloración.
Ejemplo 1: ¿Es posible cubrir un tablero de 10×10 con piezas del siguiente estilo sin que se
traslapen ni se salgan del tablero?

Nota: Las fichas se pueden voltear y girar.

Solución: Coloreamos las columnas de negro y blanco alternadamente. Entonces hay 50 casillas
blancas y 50 casillas negras. Podemos ver que sin importar cómo estén colocadas las fichas, siempre
ocupan tres casillas de un color y una casilla del otro. Llamémosles fichas de tipo 1 si ocupan tres
casillas negras y una blanca y fichas del tipo 2 si ocupan una casilla negra y tres blancas. Entonces
el número de fichas del tipo 1 debe ser igual al número de fichas del tipo 2. Con esto el número de
fichas debe ser par, por lo que el número de casillas debe ser múltiplo de 8. Como hay 100 casillas
no es posible dar la cubierta.

Ejemplo 2: En un tablero de 9×9 se colocan 65 insectos cada uno en el centro de una casilla.
Los insectos comienzan a moverse al mismo tiempo y a la misma velocidad a una casilla que com-
parte lado con la casilla en la que están. Al llegar a la siguiente casilla dan un giro de 90o y siguen
con su camino (sin salirse del tablero). Demuestra que en algún momento hay dos insectos en la
misma casilla.

Nota: Los giros pueden ser a la derecha o a la izquierda.

Solución: Vamos a colorear el tablero con 4 colores A, B, C y D de la siguiente manera:

Entonces hay 25 casillas de color A, 20 de color C, 20 de color B y 16 de color D. Fijémonos que


por el principio de las casillas, debe haber al menos 33 insectos en las casillas con colores A o D, o
33 insectos en las casillas con colores B o C. Si se da la segunda, después de un paso habrá al menos

43
33 insectos en los colores A o D. Sabemos que como los insectos tienen que ir girando, cualquier
insecto que se encuentra en una casilla con color A, después de dos pasos, pasa a una casilla con
color D. Por el principio de las casillas debe haber al menos 17 insectos en casillas de color A o al
menos 17 insectos en las casillas de color D. Si se da la primera, después de dos movidas se da la
segunda. Entonces sabemos que en algún momento debe haber al menos 17 insectos en las casillas
de color D. Luego, de nuevo por el principio de las casillas, hay dos insectos en una misma casilla.

Ejemplo 3: (Lista Corta para la IMO 2002) Un tablero de (2n − 1) × (2n − 1) se quiere
cubrir con fichas del siguiente estilo sin que se traslapen ni se salgan del tablero

Demuestra que se usan al menos 4n − 1 fichas del primer tipo.

Solución: Numeramos las columnas y las filas del 1 al 2n − 1. Vamos a colorear de negro las
casillas que esten en una columna impar y una fila impar y de blanco las demás casillas. Entonces
hay n2 casillas negras y 3n2 − 4n + 1 casillas blancas.Las fichas del primer tipo pueden cubrir 2
casillas blancas y 1 casilla negra o 3 casillas blancas. Las demás fichas cubren siempre 3 casillas
blancas y 1 casilla negra. Sea A el número de fichas del primer tipo que ocupan 1 casilla negra y
2 blancas, B el número de fichas del primer tipo que cubren 3 casillas blancas y C el número de
fichas de los otros tipos.

Contando el número de fichas negras se tiene que n2 = A + C y contando las fichas blan-
cas se tiene que 3n2 − 4n + 1 = 2A + 3B + 3C. Entonces, 4n − 1 = 3n2 − (3n2 − 4n + 1) =
3(A + C) − (2A + 3B + 3C) = A − 3B ≤ A + B. Entonces hay al menos 4n − 1 fichas del primer
tipo.

En el ejemplo anterior se usa una coloración muy especial. Es interesante notar que coloreando
como ajedrez no se obtiene algo de información, y coloreando alternadamente por columnas nos
permite probar que se usan al menos 2n − 1 fichas del primer tipo pero no se acerca a lo que pide
el problema. Otra dificultad del problema anterior es que cubrir un tablero impar con fichas de ese
tipo no es fácil. De hecho, para n = 1, 2, 3 el número de casillas que necesitan las fichas del primer
tipo es mayor al número de casillas en el tablero, por lo que ahı́ no hay cubierta posible. Para
n = 4 puede hacerse una cubierta utilizando una ficha del segundo tipo y las demás solo fichas del
primer tipo.

9.1.1. Ejercicios.
Indicaciones: Resuelva en forma clara y ordenada cada uno de los problemas que se le pre-
sentan, dejando constancia de sus soluciones.

44
1. Demuéstrese que para ninguna selección de signos en 1 ± 2 ± ... ± 10, se obtendrá una suma
0.

2. Demuéstrese que un tablero de ajedrez (8×8) no se puede recubrir totalmente (y sin sobre-
ponerse) con 15 tetrominós rectos y un L-tetrominó.

3. Demuéstrese que un tablero 8×8 nunca se podrá recubrir totalmente (sin sobreponerse) con
15 T-tetrominós y un tetrominó cuadrado.

4. Un nadador para entrenar realiza sesiones de entrenamientos de 3, 5 y 7 kilómetros. Su en-


trenador le recomienda entrenar un total de 35 kilómetros. ¿Podrá realizarlos en 10 sesiones?

5. Un saltamontes brinca a lo largo de una lı́nea. En su primer brinco salta 1 cm, en el segundo
2 cm, y ası́ sucesivamente. En cada salto él puede ir a la izquierda o a la derecha. Muestre
que después de 2001 saltos, el saltamontes no puede regresar al punto de partida.

6. ¿Se puede intercalar sı́mbolos “+” o “-” entre los números 1 2 3 4 5 6 7 8 9 10 11 12 de manera
que la suma sea 13?

7. Un test tiene 6 preguntas que valen sucesivamente 1, 2, 3, 4, 5 y 6 puntos. Cada alumno debe
responder las seis preguntas. Cuando responde correctamente se le suma el puntaje asignado
a la pregunta, y cuando responde incorrectamente, se le resta dicho puntaje. Por ejemplo,
un alumno que solo contestó correctamente las preguntas 1, 3 y 4 tiene un puntaje igual a
1 − 2 + 3 + 4 − 5 − 6 = −5. Si todos los alumnos tuvieron puntajes diferentes, calcular cuál es
la mayor cantidad de alumnos que participaron del test. Explicar cómo se llegá la conclusión.

8. ¿Se puede dibujar un camino cerrado a base de nueve segmentos, cada uno de los cuales
interseque a exactamente uno de los otros segmentos?

9. Un camino cerrado está hecho de 11 segmentos de lı́nea. ¿Puede una lı́nea que no contenga
a ningún vértice del camino, intersecar a cada uno de sus segmentos?

10. Las 28 fichas de dominó están acomodadas en una cadena, de manera que el número de puntos
en los extremos unidos de un par de fichas adyacentes coinciden. Si uno de los extremos de
la cadena es un número 5, ¿cuál es el número en el otro extremo de la cadena?

11. El producto de 22 enteros es igual a 1. Muestre que su suma no puede ser cero.

12. En la figura siguiente, la letra en cada vértice del hexágono puede sustituirse por el valor 1
ó −1. Determine el número de valores distintos que puede tomar la suma A + B + C + D
+ E + F + A(B)(C)(D)(E)(F).

45
13. En la expresión
1 ∗ 2 ∗ 3 ∗ 4 ∗ 5 ∗ 6 ∗ 7 ∗ 8 ∗ 9 ∗ 10
debemos reemplazar cada asterisco (*), ya sea con un signo más (+) o un signo menos (-)
para que el resultado sea igual a 25. Por ejemplo, una sustitución adecuada es 1 − 2 − 3 −
4 + 5 − 6 + 7 + 8 + 9 + 10. ¿De cuántas maneras diferentes se puede realizar el remplazo?

14. Se escogen 45 puntos a lo largo de una lı́nea AB, todos ellos fuera del segmento AB. Pruebe
que la suma de las distancias desde esos puntos al punto A no puede ser igual a la suma de
las distancias desde esos puntos al punto B.

15. Un piso rectangular es cubierto por mosaicos de 2×2 y de 1×4. Uno de estos mosaicos se
rompió y hay uno nuevo disponible del otro tipo. Muestre que no importa como se arreglen
los mosaicos, no se puede sustituir el roto por el nuevo.

9.2. Problemas dinámicos


9.2.1. Ejercicios.
Ejercicios.

1. En el tablero de 3×3 de la figura se va a jugar un juego. un movimiento permitido consiste


en escoger una de las casillas y cambiar de color (de negro a blanco y de blanco a negro)
todas las casillas que comparten al menos un vértice con ella. La casilla escogida no cambia
de color. Determinar si es posible, con movimientos permitidos, lograr que todas las casillas
queden del mismo color.

2. En las casillas de una cuadrı́cula de 2×4 están escritos los números 0 y 1 como se indica en
la figura.

1 0 0 0
0 0 0 1

Se va aplicar la siguiente operación: A dos casillas que compartan un lado se les puede sumar
o restar la misma cantidad. Aplicando esta operación las veces que se quiera, ¿es posible
llevar todos los números en la cuadrı́cula a 0’s?

3. Dada una lista de 0’s y 1’s le podemos aplicar la operación siguiente: Se escogen dos números
a y b de la lista, se borran y se agrega a la lista el número 0 si a = b, y 1 si a = 1 = b. Se
repite esta operación hasta quedarse con un solo número. Determinar cómo debe ser la lista
para terminar con un 1.

46
4. A una lista de números enteros se le aplica la siguiente operación: Cada número se cambia
por el número de veces que ese mismo número aparece en la lista. Esta operación se aplica
sucesivamente. Por ejemplo, a continuación se puso por renglones cómo irı́a cambiando la
lista de números que aparece en el primer renglón:

1 4 7 1 4 2 3 2 2 1 1 5
4 2 1 4 2 3 1 3 3 4 4 1
4 2 3 4 2 3 3 3 3 4 4 3
4 2 6 4 2 6 6 6 6 4 4 6
4 2 6 4 2 6 6 6 6 4 4 6

Probar que en algún momento la lista queda invariante (es decir, permanece sin cambios,
como ocurre en el ejemplo a partir del cuarto paso).

47
Capı́tulo 10
10. Recurencia
10.1. Ejemplos.
Ejemplo 13 Nelson tiene que subir una escalera que tiene 7 escalones. ¿De cuántas maneras puede
hacerlo si en cada paso puede subir uno o dos escalones? ¿De cuántas maneras puede hacerlo si la
escalera tiene n escalones?

Solución
Podemos hacerle un rápido análisis a la situación, Nelson tiene muchas formas de hacer el recorrido.
Por ejemplo podrı́a hacerlo siempre de a uno, o bien subir 3 veces de a dos y una de a uno, dos de a
dos y tres de a uno, etc. Más aún una vez determinada la cantidad de veces en los que ascenderá dos,
debe decidir cuales son los escalones en cada paso, por ejemplo si decide que ascenderá con 3 pasos
de a dos las formas en que puede hacerlo serán 1222, 2122, 2212 y 2221, en el primer caso se tiene
que primero sube un escalón y el resto los sube de dos en dos. Podemos hacer este análisis para
cada uno de los casos, es decir, primero fijar cuántos pasos de un escalón se dará y luego seleccionar
en que momentos dará esos pasos, por lo que para 7 escalones la cantidad de maneras de subir será

C41 + C53 + C65 + C77

Ahora analicemos una forma distinta de resolver el problema. Olvidémonos por un momento que
se trata de 7 escalones, y pensemos el mismo problema ahora con n escalones. Designemos por
E(n) la cantidad de formas de subir n escalones de la forma requerida, es decir, el número que
queremos encontrar.

Si analizamos el último movimiento de Nelson observamos que tenemos dos posibles situacio-
nes: o bien subió un escalón, lo que significa que se encontraba en el escalón n − 1, o bien subió dos
escalones, en cuyo caso estaba en el escalón n − 2. Estos casos son excluyentes y cubren todas las
posibilidades por lo que el número de formas de subir al escalón n es el número de formas de subir
al escalón n − 1 más el número de formas de subir al escalón n − 2. Según nuestra notación tenemos

E(n) = E(n − 1) + E(n − 2).

Luego si sabemos como empezar podemos calcular E(n), pero en nuestro caso tenemos que
E(1) = 1 y E(2) = 2, es decir hay una forma de de llegar al escalón uno y dos formas de lle-
gar al escalón dos.

Una vez teniendo esto podemos encontrar la solución a nuestro problema, el cual es encontrar E(7)
para ello utilicemos nuestra fórmula. Primero encontremos E(3) = E(2)+E(1) = 2+1 = 3, luego de
igual forma tengo que E(4) = E(3)+E(2) = 5, E(5) = E(4)+E(3) = 8, E(6) = E(5)+E(4) = 13,
de aquı́ tenemos que
E(7) = E(6) + E(5) = 21.

48
10.2. Ejercicios.
1. ¿Cuántas cadenas de ceros y unos de longitud 6, no tienen ceros consecutivos? ¿Cuántas
cadenas de ceros y unos de longitud n, no tienen ceros consecutivos?

2. ¿Cómo se puede calcular el número de subconjuntos que posee un conjunto con n elementos,
conociendo cuántos subconjuntos tiene un conjunto con n − 1 elementos? Argumente su
solución.

3. La torre de Hanoi: Se dispone de n discos de diferentes medidas y de tres clavijas en donde


estos pueden ser colocados. Inicialmente, todos los discos están colocados en una de las clavi-
jas y ordenados de abajo hacia arriba de mayor a menor. El problema consiste en trasladarlos
a otra de las clavijas siguiendo las reglas siguientes:

a) Los discos se mueven uno por uno.


b) En ningún caso puede colocarse un disco sobre otro disco de radio menor. ¿Cuál es el
número mı́nimo de movimientos requeridos para pasar todos los discos de una clavija a
otra? Primero soluciona el problema para n = 1, 2, 3, 4, 5

4. ¿Cómo se puede calcular el número de diagonales de un polı́gono de n lados inscrito en una


circunferencia, conociendo cuántas diagonales tiene un polı́gono de n − 1 lados inscrito en
una circunferencia? Argumente su solución.

5. Un alfabeto de 6 letras se codifica usando los siguientes códigos:


0 1 00 01 10 11
Se codifica una palabra sin dejar espacio entre los códigos correspondientes a cada letra. Su-
pongamos que una palabra codificada tiene 18 sı́mbolos 0 ó 1. ¿De cuántas maneras puede ser
decodificada? Por ejemplo, un código con tres sı́mbolos, tal como 011, puede ser decodificada
de tres maneras: 0/1/1, 01/1 ó 0/11

6. Determinar el máximo número de regiones determinadas por n rectas en el plano si no hay


dos de ellas que sean paralelas.

49
Guı́as Extra

50
51
Ministerio de Educación
Universidad de El Salvador
Jóvenes Talento de El Salvador
FUTUROS DIRIGENTES TÉCNICOS
CIENTÍFICOS DE EL SALVADOR
Nivel VI
4 de diciembre de 2016

Semana 1. Corto 1: Principios básicos de conteo

Nombre: Nota:

Indicaciones: Resuelva en forma clara y ordenada cada uno de los problemas que se le presentan, dejando
constancia de sus soluciones. El examen es individual, puede consultar con el equipo didáctico únicamente.

1. ¿Cuántos caminos diferentes llevan de la ciudad A a la ciudad D? (solo se puede avanzar hacia la derecha)

2. ¿De cuántas formas se pueden colocar dos alfiles, uno blanco y otro negro sobre un tablero de ajedrez de
modo que no se ataquen el uno al otro? (los alfiles se mueven en diagonal)

Crédito extra:
La distancia entre dos ciudades A y B es de 9999 kilómetros. A lo largo de la carretera, que une a estas ciudades,
hay postes indicadores de los kilómetros, en los que están escritas las distancias (en números enteros) hasta A y
hasta B. ¿Cuántos de esos postes cumplen que sólo aparezcan dos dı́gitos distintos? (Tome en cuenta que en el
primer poste aparece (0; 9999) y en el último poste, (9999; 0).)
Ministerio de Educación
Universidad de El Salvador
Jóvenes Talento de El Salvador
FUTUROS DIRIGENTES TÉCNICOS
CIENTÍFICOS DE EL SALVADOR
Nivel VI
6 de Diciembre del 2016

Semana 1. Corto 2: Caminos y combinaciones

Nombre: Nota:

Indicaciones: Resuelva en forma clara y ordenada cada uno de los problemas que se le presentan, dejando
constancia de sus soluciones

1. En las figuras siguientes:

a) ¿Cuántos caminos de longitud mı́nima llevan de A a B?


b) ¿Cuántos caminos de longitud mı́nima llevan de A a B y regresan a A?
c) ¿Cuántos caminos de longitud mı́nima llevan de A a B pasando por M o N?

2. ¿Cuántos caminos (consistiendo de una sucesión de segmentos verticales u horizontales y cada segmento
uniendo un par adyacente de letras) en las figuras siguientes deletrean CONTEST?
a) b)
C C
O C C O C
N O C C O N O C
T N O C C O N T N O C
E T N O C C O N T E T N O C
S E T N O C C O N T E S E T N O C
T S E T N O C C O N T E S T S E T N O C

Crédito extra:

Demostrar utilizando caminos la identidad


r+k+1 r+1 r+2 r+k
! ! ! ! !
r
= + + + ... + ,
k 0 1 2 k

para cualesquiera r, k ∈ Z+ .
Ministerio de Educación
Universidad de El Salvador
Jóvenes Talento de El Salvador
FUTUROS DIRIGENTES TÉCNICOS
CIENTÍFICOS DE EL SALVADOR
Nivel VI
5 de diciembre de 2016

Semana 1. Corto 3: Combinaciones

Nombre: Nota:

Indicaciones: Resuelva en forma clara y ordenada cada uno de los problemas que se le presentan, dejando
constancia de sus soluciones. El examen es individual, puede consultar con el equipo didáctico únicamente.

1. De un grupo de 10 niños y 15 niñas se quiere formar un grupo de 5 jóvenes. ¿De cuántas maneras puede
hacerse si

a) no hay más restricciones?


b) debe haber a lo sumo 2 niños?
c) no hay más restricciones para la selección, pero debe haber un(a) coordinador(a) de grupo?

2. En un torneo de basquetbol compiten 16 equipos. En cada ronda los equipos se dividen en grupos de 4. En
cada grupo cada equipo juega una vez contra cada uno de los equipos restantes. De cada grupo los mejores
dos equipos clasifican para la siguiente ronda y los dos peores son eliminados. Después de la última ronda
quedan dos equipos que se enfrentan en un partido para determinar al ganador del torneo. ¿Cuántos partidos
se jugarán a lo largo del torneo? (No importa quiénes los jugaron)

Crédito extra:
Se desea construir una escalera para ir del punto A al punto B, siendo 4.5 m la distancia de A a C y 1.5 metros
la distancia de C a B. La altura de cada escalón debe ser de 30 cm y su ancho quieren que sea un múltiplo entero de
50 cm. ¿De cuántas maneras se puede construir la escalera? Las figuras 1 y 2 muestran dos maneras de construirla.

Figura 1: Ejemplo de escalera

Figura 2: Ejemplo de escalera


Ministerio de Educación
Universidad de El Salvador
Jóvenes Talento de El Salvador
FUTUROS DIRIGENTES TÉCNICOS
CIENTÍFICOS DE EL SALVADOR
Nivel VI
7 de diciembre de 2014

Semana 1. Corto 4: Caminos

Nombre: Nota:

Indicaciones: Resuelva en forma clara y ordenada cada uno de los problemas que se le presentan, dejando
constancia de sus soluciones. El examen es individual, puede consultar con el equipo didáctico únicamente.

1. De un conjunto de 10 botes de distintos colores se quieren escoger 5 de tal manera que 3 sean para dulces y
2 sean para chocolates.¿ De cuantas formas distintas es posible hacer la elección?

2. Encuentre el número de palabras distintas que pueden obtenerse permutando las letras de la palabra MO-
MENTANEAMENTE si la primer letra debe ser M u O y no pueden estar las 4 letras E juntas.

Crédito extra:
La figura representa una telaraña en la que cada segmento mide 1. La araña se encuentra en el centro y quiere
llegar a la orilla caminando por los lados de los triángulos y usando sólo 4 segmentos en total. ¿Cuántos caminos
distintos puede seguir?
Ministerio de Educación
Universidad de El Salvador
Jóvenes Talento de El Salvador
FUTUROS DIRIGENTES TÉCNICOS
CIENTÍFICOS DE EL SALVADOR
Nivel VI
12 de diciembre de 2016

Semana 2. Corto 5: Principio de Inclusión-Exclusión y desórdenes

Nombre: Nota:

Indicaciones: Resuelva en forma clara y ordenada cada uno de los problemas que se le presentan, dejando
constancia de sus soluciones. El examen es individual, puede consultar con el equipo didáctico únicamente.

1. En una reunión hay 4 matrimonios. Se quieren tomar una foto con las 4 mujeres sentadas en un sofá y los
4 maridos parados atrás de ellas pero de forma tal que ningún hombre esté detrás de su respectiva esposa.
¿Cuántas fotos distintas se pueden tomar? (Argumente su respuesta)

2. ¿Cuántos números naturales menores que 1000 tienen la propiedad de ser divisibles entre 12, pero no divisi-
bles entre 5 ni entre 8?

Crédito extra:
¿De cuántas maneras diferentes puede distribuirse n personas en tres equipos si no necesariamente deben tener el
mismo número de integrantes pero en cada equipo debe haber al menos una persona?
Ministerio de Educación
Universidad de El Salvador
Jóvenes Talento de El Salvador
FUTUROS DIRIGENTES TÉCNICOS
CIENTÍFICOS DE EL SALVADOR
Nivel VI
13 de diciembre de 2016

Semana 2. Corto 6: Combinaciones con grupos de objetos indistinguibles. Separadores.

Nombre: Nota:

Indicaciones: Resuelva en forma clara y ordenada cada uno de los problemas que se le presentan, dejando
constancia de sus soluciones. El examen es individual, puede consultar con el equipo didáctico únicamente.

1. Andrea va a comprar jugos al supermercado donde tienen de piña, melocotón, pera y manzana. ¿De cuántas
maneras puede seleccionar los jugos si cada uno vale 40 centavos de dólar y quiere comprar todos los que
pueda con 6 dólares? ¿Y si no quiere llevar más de 10 de manzana?

2. ¿En cuántas de las permutaciones del número 23814425 aparecen los dı́gitos impares en forma creciente de
izquierda a derecha?

Crédito extra:
¿Cuántas soluciones en enteros positivos tiene la ecuación x1 x2 x3 x4 = 2737152?
Ministerio de Educación
Universidad de El Salvador
Jóvenes Talento de El Salvador
FUTUROS DIRIGENTES TÉCNICOS
CIENTÍFICOS DE EL SALVADOR
Nivel VI
14 de diciembre de 2016

Semana 2. Corto 7:Separadores.

Nombre: Nota:

Indicaciones: Resuelva en forma clara y ordenada cada uno de los problemas que se le presentan, dejando
constancia de sus soluciones. El examen es individual, puede consultar con el equipo didáctico únicamente.

1. Si se tiran 12 dados idénticos, con sus caras etiquetadas con los números del 1 al 6, ¿cuántos resultados
diferentes se pueden obtener?

2. ¿Cuántas soluciones en enteros positivos tiene la ecuación x1 + x2 + x3 + x4 + x5 + x6 = 44, si debe cumplir


que sea x2 ≥ 5 y x5 = 7? ¿y si además debe ser x1 ≤ 10 y x6 < 10?

Crédito extra:
Sean n y k enteros positivos tales que k ≤ n. ¿De cuántas formas se pueden elegir k intervalos de enteros en el
conjunto {1, 2, . . . , n} de manera que la intersección de cualesquiera dos de ellos sea vacı́a.
Nota: Un intervalo de enteros es un conjunto de uno o más enteros consecutivos.
Ministerio de Educación
Universidad de El Salvador
Jóvenes Talento de El Salvador
FUTUROS DIRIGENTES TÉCNICOS
CIENTÍFICOS DE EL SALVADOR
Nivel VI
21 de diciembre de 2016

Semana 3. Corto 8: Principio de casillas.

Nombre: Nota:

Indicaciones: Resuelva en forma clara y ordenada cada uno de los problemas que se le presentan, dejando
constancia de sus soluciones.

1. Una caja contiene 900 tarjetas numeradas del 100 al 999. Se extraen, al azar y sin reposición, tarjetas de la
caja y se anota la suma de los dı́gitos de cada tarjeta extraı́da. ¿Cuál es la menor cantidad de tarjetas que debe
extraerse para garantizar que al menos tres de esas sumas sean iguales?
¡Cuidado!, la respuesta correcta no es 55.

2. Prueba que de 12 enteros positivos distintos de dos dı́gitos, podemos seleccionar dos tales que al restarlos se
obtiene un número de dos dı́gitos de la forma aa (es decir, con dos dı́gitos iguales).

Crédito extra:
Demuestre que si se tienen 13 puntos en el plano con coordenadas enteras hay 3 de ellos tales que su baricentro
tiene coordenadas enteras. (Sugerencia: Demuestre, por cualquier método, que dados 5 enteros cualesquiera, hay 3
de ellos cuya suma es múltiplo de 3).
Ministerio de Educación
Universidad de El Salvador
Jóvenes Talento de El Salvador
FUTUROS DIRIGENTES TÉCNICOS
CIENTÍFICOS DE EL SALVADOR
Nivel VI
19 de Diciembre del 2016

Semana 3. Corto 1: Juegos de estrategia

Nombre: Nota:

Indicaciones: Resuelva en forma clara y ordenada cada uno de los problemas que se le presentan, dejando
constancia de sus soluciones

1. A y B van a jugar un juego por turnos. Antes de empezar, se ponen alrededor de un cı́rculo junto con otras
2001 personas. En cada turno se vale sacar del cı́rculo a alguno de sus vecinos. Entre A y B gana quien logre
sacar a la otra persona. Si A empieza, decide quien tiene estategia ganadora. Nota: Las demás 2001 personas
no tienen turnos.

Crédito extra:

El número 3 se encuentra escrito en una pizarra. Piche y Erick juegan por turnos, empieza Piche y juegan de
la siguiente forma: Si el número escrito en la pizarra es n el jugdor en su turno lo debe remplazar por un entero
m coprimo con n y tal que n < m < n2 . El primer jugador en llegar a un número mayor o igual que 2016 pierde.
Determine quien tiene la estrategia ganadora y describala.
Ministerio de Educación
Universidad de El Salvador
Jóvenes Talento de El Salvador
FUTUROS DIRIGENTES TÉCNICOS
CIENTÍFICOS DE EL SALVADOR
Nivel VI
20 de diciembre de 2016

Semana 3. Corto 10: Coloración y paridad.

Nombre: Nota:

Indicaciones: Resuelva en forma clara y ordenada cada uno de los problemas que se le presentan, dejando
constancia de sus soluciones. El examen es individual, puede consultar con el equipo didáctico únicamente.

* Problema. Benjamı́n tiene 50 piezas, de las cuales hay 25 de cada uno de los siguientes dos tipos:

Utilizando esas piezas, él desea cubrir tableros sin dejar huecos, sin superponer piezas y sin salirse del tablero.
Las piezas se pueden girar e incluso voltear.

a) ¿Podrá cubrir un tablero de 8×9?


b) ¿Podrá cubrir un tablero de 9×9?
c) ¿Podrá cubrir la siguiente figura, la cuál se obtiene al quitar todas las casillas por arriba de la diagonal
principal de un tablero de 8×8?

Crédito extra:
Alrededor de una mesa redonda están sentadas en sentido horario las personas P1 , P2 , . . . , P2013 . Cada una tiene
cierta cantidad de monedas (posiblemente ninguna); entre todas tienen 10000 monedas. Comenzando por P1 y
prosiguiendo en sentido horario, cada persona en su turno hace lo siguiente:

Si tiene un número par de monedas, se las entrega todas a su vecino de la izquierda.

Si en cambio tiene un número impar de monedas, le entrega a su vecino de la izquierda un número impar de
monedas (al menos una y como mx́imo todas las que tiene), y conserva el resto.

Pruebe que, repitiendo este procedimiento, necesariamente llegará un momento en que todas las monedas estén en
poder de una misma persona.
Ministerio de Educación
Universidad de El Salvador
Jóvenes Talento de El Salvador
FUTUROS DIRIGENTES TÉCNICOS
CIENTÍFICOS DE EL SALVADOR
Nivel VI
17 de diciembre de 2013

Semana 2. Corto 5.

Nombre: Nota:

Indicaciones: Resuelva en forma clara y ordenada cada uno de los problemas que se le presentan, dejando
constancia de sus soluciones.

1. Andrea va a comprar jugos al supermercado donde tienen de piña, melocotón, pera y manzana. ¿De cuántas
maneras puede seleccionar los jugos si cada uno vale 40 centavos de dólar y quiere comprar todos los que
pueda con 6 dólares? ¿Y si no quiere llevar más de 4 de manzana?

2. ¿Cuántas soluciones en enteros positivos tiene la ecuación x1 + x2 + x3 + x4 + x5 + x6 = 44, si debe cumplir


que x2 ≥ 5, x5 = 7, x1 ≤ 10 y x6 < 10?

Crédito extra:
¿Cuántas soluciones en enteros positivos tiene la ecuación x1 x2 x3 x4 = 2737152?
Ministerio de Educación
Universidad de El Salvador
Jóvenes Talento de El Salvador
FUTUROS DIRIGENTES TÉCNICOS
CIENTÍFICOS DE EL SALVADOR
Nivel VI
22 de Diciembre del 2016

Semana 3. Corto. 4 Recurrencia

Nombre: Nota:

Indicaciones: Resuelva en forma clara y ordenada cada uno de los problemas que se le presentan, dejando
constancia de sus soluciones

1. Mynor quiere subir una escalera con 9 escalones. ¿De cuántas formas puede hacerse si se pueden dar pasos
de longitud 1, 2, o 3?

2. ¿De cuántas formas se puede cubrir un tablero rectangular de dimensión 2 × n con piezas de dimensiones
1 × 2 y 2 × 2? Utiliza lo anterior para calcular el número de formas si n = 9.

Crédito extra:
¿De cuántas formas se puede cubrir un tablero de tamaño 3 × 2n con piezas de tamano 1 × 2?
Ministerio de Educación
Universidad de El Salvador
Jóvenes Talento de El Salvador
FUTUROS DIRIGENTES TÉCNICOS
CIENTÍFICOS DE EL SALVADOR
Nivel VI
4 de diciembre de 2016

Semana 1. Tarea 1: Principios básicos de conteo y combinaciones

Indicaciones: Resuelva en forma clara y ordenada cada uno de los problemas que se le presentan, dejando
constancia de sus soluciones.

1. ¿Cuántas maneras distintas hay de colocar un rey blanco y uno negro en un tablero de ajedrez de modo que
no se ataquen entre sı́?

2. Se escriben los enteros del 1 al 1000 en sucesión. Hállese la suma de todos los dı́gitos.

Crédito extra:
Dentro de los cuadros de una cuadrı́cula de 22×22 se escriben los números 1, 2, 3, . . . , 400 y ochenta y cuatro veces
el número 0 de la manera siguiente: Los números 0 se escriben en todos los cuadros de la orilla y después en forma
espiral se van escribiendo sucesivamente los números 1, 2, 3, . . . , 400 como se indica en la figura.

Luego, en cada vértice interior de la cuadrı́cula hay que poner la suma de todos los números que aparecen en los
cuadrados que forman el vértice, por ejemplo, hay que poner 156 y 5, respectivamente en los vértices centrales de:

Calcular la suma de todos los números escritos en los vértices.


Ministerio de Educación
Universidad de El Salvador
Jóvenes Talento de El Salvador
FUTUROS DIRIGENTES TÉCNICOS
CIENTÍFICOS DE EL SALVADOR
Nivel VI
5 de diciembre de 2016

Semana 1. Tarea 2

Nombre: Nota:

Indicaciones: Resuelva en forma clara y ordenada cada uno de los problemas que se le presentan, dejando
constancia de sus soluciones.

1. Para la siguiente figura, cuántos caminos de longitud mı́nima

a) Llevan de A a X.
b) Llevan de A a Y.
c) Llevan de A a B pasando por X.
d) Llevan de A a B.
e) Llevan de A a B pasando por X o por Z.

2. ¿De cuántas formas se pueden colocar dos alfiles, uno blanco y otro negro sobre un tablero de ajedrez?

a) De modo que no se ataquen el uno al otro


b) ¿Cual seria la respuesta de a) si los alfiles son ambos negros?

Crédito extra:
Demuestra que si n es un entero positivo, entonces
! !2 !2 !2 !2
2n n n n n
= + + + ... + .
n o 1 2 n

Debes demostrarlo de dos maneras: con un argumento contando caminos en una cuadrı́cula de n × n y con un
argumento que consista en contar de dos maneras diferentes las formas de seleccionar n objetos o personas de un
grupo de 2n disponibles.
Ministerio de Educación
Universidad de El Salvador
Jóvenes Talento de El Salvador
FUTUROS DIRIGENTES TÉCNICOS
CIENTÍFICOS DE EL SALVADOR
Nivel VI
6 de diciembre de 2016

Semana 1. Tarea 3: Permutaciones

Indicaciones: Resuelva en forma clara y ordenada cada uno de los problemas que se le presentan, dejando cons-
tancia de sus soluciones.

1. Para jugar mafı́a se disponen 20 personas en cı́rculo y se asignan los siguientes roles: 5 asesinos, 1 policı́a
y 1 médico, mientras el resto de personas juegan como civiles. Determine el número de formas en que se
pueden sentar a jugar (si ya se sabe quien es el policı́a, quien es el médico y quienes son los asesinos) si:

a) No importa el orden.
b) Todos los asesinos quedan juntos, mientras que el médico y el policı́a quedan separados.

2. Un tablero cuadrado de 8cm de lado se divide en 64 casillas cuadrados de 1cm de lado cada una. Cada casilla
se puede pintar de blanco o de negro. Encuentra el número total de maneras de colorear el tablero de modo
que cada cuadrado de 2cm de lado formado por cuatro casillas con un vértice común, contenga dos casillas
blancas y dos casillas negras.
 
Crédito extra: Sea p un número primo, demuestra que 2p 2
p − 2 es múltiplo de p .
Ministerio de Educación
Universidad de El Salvador
Jóvenes Talento de El Salvador
FUTUROS DIRIGENTES TÉCNICOS
CIENTÍFICOS DE EL SALVADOR
Nivel VI
7 de diciembre de 2016

Tarea 4

Nombre: Nota:

Indicaciones: Resuelva en forma clara y ordenada cada uno de los problemas que se le presentan, dejando
constancia de sus soluciones.

Problema 1
Demuestre (algebraicamente) la identidad
k2 = 2Ck2 + Ck1
y a partir de ella demuestre que !
1 1
1 + 2 + · · · + n = n n + (n + 1).
2 2 2
3 2

Problema 2
Una araña tiene 8 pies, 8 zapatos distintos y 8 calcetines distintos. Encuentra el número de maneras en que puede
ponerse los 8 calcetines y los 8 zapatos (considerando el orden en que se los pone), con la condición de que antes
de ponerse un zapato tiene que tener ya un calcetı́n es ese pie.

Crédito Extra:
En todas las casillas de la pirámide siguiente se escribe un número entero positivo de tal manera que para cada
pareja de números en casillas consecutivas en la misma fila, su producto es igual al número en la casilla adyacente
a ambas en la fila superior. Si el número ubicado en la casilla superior es 64 y la pirámide tiene seis filas, ¿cuál es
la menor cantidad de casillas con el número 1 que se pueden tener?
Ministerio de Educación
Universidad de El Salvador
Jóvenes Talento de El Salvador
FUTUROS DIRIGENTES TÉCNICOS
CIENTÍFICOS DE EL SALVADOR
Nivel VI
11 de diciembre del 2016

Semana 2. Tarea1

Nombre: Nota:

Indicaciones: Resuelva en forma clara y ordenada cada uno de los problemas que se le presentan, dejando
constancia de sus soluciones.

Problema 1
Un grupo de 102 estudiantes se examinan en Matemática, Sociales y Lenguaje. De entre ellos, 92 pasaron ma-
temática, 75 sociales y 63 Lenguaje, 65 pasaron Matemática y sociales, 54 Matemática y Lenguaje y 48 Sociales y
Lenguaje. ¿Cuántos estudiantes pasaron las tres materias? (Ningún estudiante reprobó las tres materias)

Problema 2
En una fiesta, cuatro amigos se van a dar regalos entre sı́, de manera que cada uno de un regalo y reciba otro,
desde luego nadie debe darse el regalo a sı́ mismo (algo ası́ como si jugaran Amigo Secreto). ¿De cuántas formas
es posible hacer la distribución?

∞∞∞∞∞∞∞∞∞∞∞∞∞∞∞∞∞∞∞∞∞∞∞∞∞∞∞∞∞∞∞∞∞∞∞∞∞∞∞∞∞∞∞∞∞∞∞∞∞∞∞∞
Y porque usted lo pidió, se queda, úuuuuuuuultimo dı́a:
Problema 2 (tarea anterior)
Una araña tiene 8 pies, 8 zapatos distintos y 8 calcetines distintos. Encuentra el número de maneras en que puede
ponerse los 8 calcetines y los 8 zapatos (considerando el orden en que se los pone), con la condición de que antes
de ponerse un zapato tiene que tener ya un calcetı́n es ese pie.

∞∞∞∞∞∞∞∞∞∞∞∞∞∞∞∞∞∞∞∞∞∞∞∞∞∞∞∞∞∞∞∞∞∞∞∞∞∞∞∞∞∞∞∞∞∞∞∞∞∞∞∞
Crédito Extra:
Una pelota de fútbol esta formada de piezas de cuero blancas y negras. Las piezas negras son pentágonos regu-
lares y las piezas blancas hexágonos regulares. Cada pentágono esta rodeada por 5 hexágonos y cada hexágono
esta rodeado por 3 pentágonos y 3 hexágonos. La pelota tiene 12 pentágonos negros. ¿Cuántos hexágonos blancos
tiene?
Ministerio de Educación
Universidad de El Salvador
Jóvenes Talento de El Salvador
FUTUROS DIRIGENTES TÉCNICOS
CIENTÍFICOS DE EL SALVADOR
Nivel VI
12 de diciembre de 2016

Semana 2. Tarea 2 - Separadores.

Nombre: Nota:

Indicaciones: Resuelva en forma clara y ordenada cada uno de los problemas que se le presentan, dejando
constancia de sus soluciones.

Problema 1
José comprará 20 galletas de distintos sabores: fresa, chocolate, vainilla y limón. ¿De cuántas formas puede hacer
la compra si tiene que llevar un número de galletas de vainilla que sea 4 veces el número de galletas de limón?

Problema 2
¿De cuántas maneras pueden seleccionarse 11 letras x, y ó z (no importa el orden) si todas las letras deben aparecer
al menos dos veces y a lo sumo 5?

Crédito Extra:
Probar que si ki=1 ai ≤ n, (los ai ∈ Z+ ), entonces a1 !a2 ! . . . ak ! es un divisor de n!.
P
Ministerio de Educación
Universidad de El Salvador
Jóvenes Talento de El Salvador
FUTUROS DIRIGENTES TÉCNICOS
CIENTÍFICOS DE EL SALVADOR
Nivel VI
13 de diciembre de 2016

Semana 2. Tarea 3: Separadores

Indicaciones: Resuelva en forma clara y ordenada cada uno de los problemas que se le presentan, dejando con-
stancia de sus soluciones.

1. Determine el coeficiente de x2 yz9 en el desarrollo de (1 + x + y + z + w)15 .

2. Encuentre el número de cuaternas (x1 , x2 , x3 , x4 ) de enteros positivos impares tales que

x1 + x2 + x3 + x4 = 98.

Crédito extra: Los Doce Caballeros se reunen en la Mesa Redonda del Rey Arturo a discutir el rescate de una
princesa. Cada caballero ve a sus dos vecinos en la mesa como enemigos. ¿De cuántas formas puede escogerse un
equipo de cinco caballeros para el rescate de modo que no contenga enemigos?
Ministerio de Educación
Universidad de El Salvador
Jóvenes Talento de El Salvador
FUTUROS DIRIGENTES TÉCNICOS
CIENTÍFICOS DE EL SALVADOR
Nivel VI
14 de diciembre de 2016

Semana 2. Tarea 4

Nombre: Nota:

Indicaciones: Resuelva en forma clara y ordenada cada uno de los problemas que se le presentan, dejando
constancia de sus soluciones.

Problema 1
Se tienen 512 bloques cúbicos y se han apilado todos formando un cubo más grande. Una hormiga está en un vérti-
ce del cubo grande y camina hasta el vértice opuesto (un vértice de un cubo corresponde a tres caras y el vértice
opuesto es el que no corresponde a esas tres caras) pero solo se mueve siguiendo los caminos que determinan las
aristas de los cubos. ¿Cuántos caminos diferentes de la mı́nima longitud puede seguir la hormiga?

Problema 2
Encuentre el número de enteros positivos x tales que x ≤ 9999999 y la suma de sus dı́gitos sea igual a 31.

Crédito Extra:
Probar que si ki=1 ai ≤ n, (los ai ∈ Z+ ), entonces a1 !a2 ! . . . ak ! es un divisor de n!.
P
Ministerio de Educación
Universidad de El Salvador
Jóvenes Talento de El Salvador
FUTUROS DIRIGENTES TÉCNICOS
CIENTÍFICOS DE EL SALVADOR
Nivel VI
16 de diciembre de 2016

Semana 3. Tarea 9: Juegos de estrategia

Indicaciones: Resuelva en forma clara y ordenada cada uno de los problemas que se le presentan, dejando cons-
tancia de sus soluciones.

1. Las cifras de una calculadora (a excepción del 0) están dispuestas en la forma indicada en el cuadro adjunto,
donde aparece también la tecla “+”.

Dos jugadores A y B juegan de la manera siguiente: A enciende la calculadora y pulsa una cifra, y a con-
tinuación pulsa la tecla +. Pasa la calculadora a B , que pulsa una cifra en la misma fila o columna que la
pulsada por A que no sea la misma que la última pulsada por A; a continuación pulsa + y le devuelve la
calculadora a A, que repite la operación y ası́ sucesivamente. Pierde el juego el primer jugador que alcanza o
supera la suma 31. ¿Cuál de los dos jugadores tiene una estrategia ganadora y cuál es ésta?

2. Dennis y Emiliano inician un juego donde alternadamente van sustituyendo el número escrito en la pizarra.
En cada turno, el jugador debe sustituir el número escrito, ya sea por la cantidad de divisores del nḿero
escrito o por la diferencia entre el número escrito y su cantidad de divisores. Dennis es el primero en jugar,
y aquel jugador que escriba el 0 gana. Dado que el nÃo mero inicial es 1036, determine cuál de los jugadores
tiene una estrategia ganadora y describa dicha estrategia.
Nota: Por ejemplo, la cantidad de divisores del 14 es 4, pues sus divisores son 1, 2, 7 y 14
Ministerio de Educación
Universidad de El Salvador
Jóvenes Talento de El Salvador
FUTUROS DIRIGENTES TÉCNICOS
CIENTÍFICOS DE EL SALVADOR
Nivel VI
19 de diciembre de 2016

Semana 3. Tarea 10: Coloración y paridad.

Indicaciones: Resuelva en forma clara y ordenada cada uno de los problemas que se le presentan, dejando con-
stancia de sus soluciones.

Problema 1
Pedro compró un cuaderno que contiene 96 hojas y numeró sus páginas del 1 al 192. Vı́ctor desprende 25 hojas del
cuaderno de Pedro y después suma los 50 números escritos en las páginas. ¿Puede Vı́ctor obtener 1990 como la
suma?

Problema 2
A un tablero cuadriculado de 9×9 se le han quitado tres de sus esquinas (de 1×1). ¿Es posible cubrirlo con fichas
de 3 x 1?

Crédito extra:
(El zorro y el ganso). Se juega en el tablero que muestra la figura

Hay que poner dos fichas distintas entre sı́, una en el lugar donde está el retrato del zorro; y otra en donde está el
retrato del ganso. Uno de los jugadores moverá al zorro, el otro moverá al ganso. Una “movida” consiste en deslizar
la ficha desde un punto hasta otro adyacente, siguiendo una de las lı́neas negras trazadas en el tablero. El zorro
intentará capturar al ganso desplazándose hacia el punto ocupado por el ganso. El ganso debe intentar impedir que
esto suceda. Si el zorro captura al ganso en diez movimientos o menos (es decir, en diez movimientos del zorro),
ganará el juego. Si no logra su objetivo, ganará el ganso. El primer movimiento del juego lo hace el zorro. ¿Alguno
de los jugadores tiene estrategia ganadora?
Ministerio de Educación
Universidad de El Salvador
Jóvenes Talento de El Salvador
FUTUROS DIRIGENTES TÉCNICOS
CIENTÍFICOS DE EL SALVADOR
Nivel VI
20 de diciembre de 2016

Semana 3. Principio de casillas

Nombre: Nota:

Indicaciones: Resuelva en forma clara y ordenada cada uno de los problemas que se le presentan, dejando
constancia de sus soluciones.

Problema 1
Los enteros del uno 1 al 10 se ponen en los vértices de un decágono. Demostrar que siempre se pueden encontrar
tres vértices seguidos cuya suma es mayor que 16.
Problema 2
En una cuadrı́cula de 8×8 se han escogido arbitrariamente 10 cuadritos y se han marcado los centros de éstos.
El lado de cada cuadrito mide 1. Demuestre que existen al menos dos puntos marcados que están separados una
√ 1
distancia menor o igual que 2, o que existe al menos un punto marcado que se encuentra a distancia de una
2
orilla de la cuadrı́cula.

Crédito Extra

1. Se tiene un tablero de 100 × 100. En cada casilla hay una flecha que apunta hacia arriba, abajo, a la derecha
o a la izquierda. En toda la orilla del tablero hay una pared excepto por el lado derecho de la casilla de la
esquina superior derecha. Se va a colocar un insecto en una de las casillas. Al encontrarse en una casilla, el
insecto se va a mover a la casilla vecina en la dirección de la flecha sobre la que está. Después de hacer esto,
la flecha sobre la que estaba rota 90 grados en el sentido de las manecillas del reloj. Si el movimiento que se
indica no se puede hacer, el insecto se queda en su lugar pero la flecha sı́ se mueve y se continúa el proceso.
¿Es posible que el insecto nunca salga del tablero?
Ministerio de Educación
Universidad de El Salvador
Jóvenes Talento de El Salvador
FUTUROS DIRIGENTES TÉCNICOS
CIENTÍFICOS DE EL SALVADOR
Nivel VI
21 de Diciembre del 2016

Semana 3. Tarea: Recurrencia

Nombre: Nota:

Indicaciones: Resuelva en forma clara y ordenada cada uno de los problemas que se le presentan, dejando
constancia de sus soluciones

1. Supongamos que se desea cubrir un rectángulo de dimension 2 × n utilizando rectángulos de dimension 2 × 1


¿De cuantas formas puede hacerse?

Crédito extra:
Se tiene un triángulo equilatero de lado n y se divide en triángulos equilateros de lado 1, cuyos lados son paralelos
al triángulo original. Encuentre la cantidad de paralelogramos que se pueden formar con los lados de los triángulos
pequeños.
Ministerio de Educación
Universidad de El Salvador
Jóvenes Talento de El Salvador
FUTUROS DIRIGENTES TÉCNICOS
CIENTÍFICOS DE EL SALVADOR
Nivel VI
Combinatoria

Fecha: Viernes, 9 de diciembre de 2016 Examen Semanal 1

Nombre Completo:

Indicaciones: Resuelva en forma clara y ordenada cada uno de los problemas que se le presentan, dejando
constancia de sus soluciones. El examen es individual, es decir soluciones idénticas serán anuladas.

* Problema 1. [25 %]

a) Las caras de un dado están numeradas de 1 a 6 y se desea pintarlas utilizando seis colores distintos,
cada cara de un color. ¿De cuántas maneras diferentes puede quedar pintado el dado?
b) En una avenida hay diez semáforos. En un momento en particular, ¿cuántas posibilidades diferentes
existen para la configuración de colores de los diez semáforos?
c) ¿De cuántas maneras distintas se pueden ordenar en fila cuatro bolitas rojas, tres azules y dos verdes?
d) Una chicago tiene diez bagones y se van a pintar usando diez colores distintos, cada bagón de un color.
¿De cuántas maneras distintas puede quedar pintados los bagones?

* Problema 2. [25 %] ¿Cuántos caminos diferentes hay, en la figura siguiente, para ir del punto M al punto N,
movı́endose siempre hacia abajo sobre las lı́neas verticales o inclinadas?

* Problema 3. [25 %] Se dispone de una colección de 30 pelotas divididas en 5 tamaños distintos y 6 colores
diferentes de tal manera que en cada tamaño hay los 6 colores. ¿Cuántas colecciones de 4 pelotas tienen
exactamente 2 pares de pelotas del mismo tamaño (que no sean todas del mismo tamaño, es decir que sean
dos de un tamaño y las otras dos de igual tamaño entre sı́ pero distinto al de las anteriores)?
* Problema 4. [25 %] (ONM 2013) Considere el siguiente arreglo de puntos sobre el tablero de la figura.

Determine el número de formas en que se pueden seleccionar tres de esos puntos, para que sean los vértices
de un triángulo rectángulo cuyos catetos sean paralelos a los lados del tablero.

Créditos extra:

1. Demuestre que
2n−1 k
X C2n
=0
k=0
(−2)k

2. Sean k, n ∈ N. Demuestre que el número de formas de sentar kn personas alrededor de k mesas distintas de
modo que hay n personas en cada mesa es
(kn)!
.
nk
(ninguna mesa sirve de referencia para las demás)
Ministerio de Educación
Universidad de El Salvador
Jóvenes Talento de El Salvador
FUTUROS DIRIGENTES TÉCNICOS
CIENTÍFICOS DE EL SALVADOR
Nivel VI
Combinatoria

Fecha: Viernes, 16 de diciembre de 2016 Examen Semanal 2

Nombre Completo:

Indicaciones: Resuelva en forma clara y ordenada cada uno de los problemas que se le presentan,
dejando constancia de sus soluciones. El examen es individual, es decir soluciones idénticas serán anuladas.

* Problema 1. [25 %] Mynor fue a la tienda a comprar dulces de diferentes sabores. En la tienda
habı́an dulces de fresa, uva, piña y miel. Los dulces de miel los vendı́an en bolsitas de 6 dulces cada
una, mientras que los dulces del resto de sabores los vendı́an individuales. Si a Mynor le encantan
los de fresa y querı́a comprar por lo menos 4 de éstos, ¿De cuántas formas distintas puede comprar
20 dulces?

* Problema 2. [25 %]Una caja contiene 900 tarjetas numeradas del 100 al 999. Se extraen, al azar y
sin reposición, tarjetas de la caja y se anota la suma de los dı́gitos de cada tarjeta extraı́da. ¿Cuál
es la menor cantidad de tarjetas que debe extraerse para garantizar que al menos tres de esas sumas
sean iguales?
¡Cuidado!, la respuesta correcta no es 55.

* Problema 3. [20 %]Contar de dos maneras diferentes la cantidad de ternas de números del 1 al n
que están en orden creciente (n es un número natural), para probar la fórmula:
       
n 2 3 n−1
= + + ... + .
3 2 2 2

* Problema 4. [20 %] Erick va todos los viernes al cine, y ha programado qué pelı́culas verá en los
próximos 5 viernes, aunque todavı́a no ha decidido en que orden. Piche, que tiene idéntica costumbre
que Erick, ha hecho lo propio. Si ambos eligieron las mismas pelı́culas, ¿De cuántas formas puede
ocurrir que ningún dı́a coincidan en la pelı́cula elegida?

* Problema 5. [20 %] Ocho cajas numeradas del 1 al 8 están colocadas en una fila en el orden de
su numeración. ¿De cuántas formas distintas pueden colocarse 8 esferas de navidad en las cajas si
de cada uno de 4 colores distintos hay dos esferas iguales, en cada caja va una esfera y esferas del
mismo color no deben quedar en cajas con numeración consecutiva?

Crédito extra:
En una fiesta hay 100 personas. La primera da la mano a 1 persona, la segunda a 2 personas, la tercera
a 3 personas y ası́ sucesivamente, de manera que la nonagésima novena da la mano a 99 personas. ¿A
cuántas personas le dio la mano la número 100?
Ministerio de Educación
Universidad de El Salvador
Jóvenes Talento de El Salvador
FUTUROS DIRIGENTES TÉCNICOS
CIENTÍFICOS DE EL SALVADOR
Nivel VI
Combinatoria

Fecha: Viernes, 23 de diciembre de 2016 Examen Semanal 3

Nombre Completo:

Indicaciones: Resuelva en forma clara y ordenada cada uno de los problemas que se le presentan, dejando
constancia de sus soluciones. El examen es individual, es decir soluciones idénticas serán anuladas.

* Problema 1. [25 %] Se tiene una cuadricula de 5 × 8. Sea A el vértice inferior izquierdo y B el vértice
superior derecho. ¿Será posible llegar de A a B en 2012 pasos moviéndose sobre las lı́neas (en cualquier
dirección)? Nota: un paso es un movimiento de igual longitud que un lado de un cuadrado pequeño.

* Problema 2. [25 %] Márquese a un disco con la etiqueta “1”, a dos discos con la etiqueta “2”, a tres discos
con la etiqueta“3”,..., a cincuenta discos con la etiqueta “50”. Póngase a éstos 1 + 2 + 3 + . . . + 50 discos en
una caja. Se sacan luego discos de la caja al azar y sin remplazo. ¿Cuál es el número mı́nimo de discos que
se debe sacar para garantizar al menos diez discos con la misma etiqueta?

* Problema 3. [25 %] Dado un cuadrado de lado 2n con n entero mayor o igual a 1. Demostrar que si se extrae
una esquina (se quita un cuadrado de lado 1), es posible cubrirlo exactamente con piezas en forma de L (cada
una con tres cuadrados de lado 1).

* Problema 4. [25 %] Juan y Marı́a juegan de la siguiente manera: Primero Juan escoge tres números del
conjunto {1, 2, 3, 4, 5, 6}, luego Marı́a escoge un número M que puede ser uno de los tres números que Juan
no escogió, o el producto de dos de los números que Juan no escogió o el producto de los tres números
que Juan no escogió. Finalmente, Juan forma un número J de tres cifras ordenando como le plazca los tres
números que escogió. Si J + M es múltiplo de 7 gana Marı́a, de lo contrario gana Juan. Determine si alguno
de ellos tiene una estrategia ganadora y explı́quela.

Crédito extra:

1. Una mesa circular tiene exactamente sesenta sillas alrededor. ¿Cuál es el número mı́nimo de personas que
pueden sentarse estratégicamente en esas sillas, de manera que la próxima persona a sentarse obligatoria-
mente se sentará al lado de alguien?

2. ¿Qué carrera estudiará cuando ingrese a la universidad?

§§§§§§§§§§§§§§§§§§§§§§§§§§§§§§

Solo alumnos ASD, por favor, escriba el nombre de la Institución Educativa en la que estudia, si es pública o
privada, rural o urbana, el grado que cursó o cursa, su ciudad y paı́s de procedencia, su fecha de nacimiento y su
dirección de correo electrónico.
UNIVERSIDAD DE EL SALVADOR

PROGRAMA JÓVENES TALENTO-MINISTERIO DE EDUCACIÓN

FUTUROS DIRIGENTES TÉCNICOS Y CIENTÍFICOS - FDTC 2016

SÍLABO - NIVEL VI

COMBINATORIA

SAN SALVADOR, EL SALVADOR, 7 DE NOVIEMBRE DE 2016.


2

1. Generalidades

1. Duración del curso: 3 semanas del 4 al 23 de diciembre.


2. Año: 2016
3. Nivel: VI
4. Personal responsable:

Mynor Ademar Melara Estrada


Licenciado en Matemática, Universidad de El Salvador
Correo: mynormelara@jovenestalento.edu.sv
myscorpionm7@gmail.com

Erick Amílcar Muñoz Deras


Estudiante de Licenciatura en Matemática, Universidad de El Salvador.
Correo: erickderas@jovenestalento.edu.sv
erickderas@hotmail.es

Carlos Ariel Piche Cruz


Bachiller técnico bilingüe y alumno olímpico de matemática del Programa Jóvenes Talento.
Correo: arielpiche2357@gmail.com

2. Descripción del curso

El curso está orientado a desarrollar la habilidad de resolver problemas propios de conteo o que
involucren el análisis de conguraciones.
se iniciará con el estudio de elementos básicos de la teoría de conjuntos como una herramienta
para el desarrollo de los demás contenidos del curso. Los primeros contenidos de Análisis Com-
binatorio que se cubrirán estarán centrados en fortalecer las técnicas básicas de conteo y en
torno a ellas girarán las deniciones de conceptos y el estudio de principios básicos, esto es, el
pricipio de la suma, el principio del producto y el principio de inclusión y exclusión, para luego
llegar al estudio de las combinaciones por medio de al menos tres enfoques: modelo conjuntista,
modelo de caminos y cadenas binarias. Se completarán luego las técnicas de conteo al estudiar
las permutaciones y las combinaciones con repetición. Los temas tratados, forman parte fun-
damental para la comprensión de contenidos desarrollados tanto en matemática como en otras
ciencias. Por ejemplo, en estadística, el cálculo de probabilidades llega a ser un pilar fundamen-
tal en donde se utilizan ampliamente los teoremas que se estudiarán, como lo son el teorema del
binomio y el principio de inclusión, así como las combinaciones y permutaciones. Continuando
con el curso se dará paso a otros tópicos centrados en el estudio de conguraciones, como el
principio de Dirichlet y los números de Ramsey, doble conteo coloración y paridad, juegos de
estrategia y principio extremo, y recurrencias.
3. Objetivo del curso

Desarrollar en el estudiante el lenguaje, el pensamiento matemático y las habilidades y des-


trezas necesarias para la resolución satisfactoria de problemas que involucren el análisis de
conguraciones, tales como los problemas de conteo, los que suelen aparecer en olimpiadas

2
3

matemáticas y los del tipo básico de Matemática Discreta. Además de la preparación para
identicar aplicaciones en diferentes áreas de la matemática y otras ciencias.
4. Contenido a desarrollar

1. ¾Qué es la combinatoria? (4 de diciembre)


2. Nociones básicas de la teoría de conjuntos (4 de diciembre, solo un resumen de lo necesario)
3. Principios básicos de conteo (4 de diciembre, esto será lo principal de la clase)
3.1 Principio de la suma (4 de diciembre)
3.2 Principio de la multiplicación (4 de diciembre)
4. Combinaciones
4.1 Modelo de conjuntos (5 de diciembre, antes de receso)
4.2 Modelo de cadenas binarias (5 de diciembre, solo aparecerá en los ejercicios)
4.3 Modelo de caminos (5 de diciembre, después de receso)
5. Permutaciones
5.1 Denición (6 de diciembre)
5.2 Factorial de un número natural (6 de diciembre)
5.3 Arreglos (6 de diciembre, antes de receso)
5.4 Permutaciones con objetos repetidos (6 de diciembre, después de receso)
5.5 Permutaciones en una circunferencia (7 de diciembre antes de receso)
6. Teorema del binomio (7 de diciembre, después de receso)
7. Principio de inclusión-exclusión y desórdenes (8 de diciembre. Hasta aquí el primer semanal)
8. Combinaciones con repetición. Separadores (Inicio segunda semana, 9, 12 de diciembre)
9. Doble conteo. (13 de diciembre)
10. Principio de Dirichlet y problemas tipo Ramsey (14, 15 de diciembre. Hasta aquí el segundo
semanal)
11. Juegos de estrategia (Inicio tercera semana, 16 de diciembre)
12. Coloración y paridad (19 de diciembre)
13. Inducción matemática (20 de diciembre)
14. Relaciones de recurrencia y funciones generatrices (21, 22 de diciembre)

5. Metodología

Se adoptará un enfoque de resolución de problemas proporcionándole al estudiante conceptos


y métodos prácticos de conteo que le permitan resolver problemas creativos. Se incentivará la
solución de problemas de alto y bajo nivel mediante la aplicación de los resultados elementales
del conteo. Se abordaran problemas que involucren un manejo algebraico introductorio y se
presentarán diferentes alternativas de solución a problemas especícos de combinatoria, siendo
el alumno quien deba escoger el camino más factible. Se hará énfasis en la demostración de
identidades a la vez que se planteará una comparación de diversos métodos de demostración
para tales identidades. Las clases se dividirán en dos momentos principales, primero se pre-
sentarán los nuevos conceptos los cuales se pretende que sean introducidos al enfrentar a los

3
4

alumnos a un nuevo problema y llegar juntos a la solución para luego formalizar el trabajo
y desarrollar diversos ejemplos complementarios en la pizarra, los cuales se trabajarán y se
llegará a la solución con la mayor participación por parte de los alumnos, y luego se entregará
a los alumnos la guía de ejercicios que deberán trabajar principalmente de manera individual
pero podrán consultarse entre ellos o a los responsables del curso y en ocasiones se pasará a
los alumnos a resolver ejercicios en la pizarra, para vericar el nivel de comprensión de los
contenidos y detectar errores para tomar medidas al respecto.
6. Evaluaciones.

Tareas:
Se asignará una tarea cada día de clase, excepto el día del examen semanal, con el n que
puedan profundizar y cimentar los conocimientos adquiridos en la clase.
Cortos:
Cada día se realizará un examen corto en el que se evaluarán las temáticas impartidas el
día anterior, cuya duración máxima será de 25 minutos y se desarrollarán antes del receso.
Examenes semanales: Esta evaluación tendrá lugar el viernes de cada una de las tres
semanas que dura el curso. Cubre el material impartido durante la semana correspondi-
ente.
Créditos extra:
Son problemas de desafío, que se pueden desarrollar utilizando los conceptos vistos en
clase, pero exigen creatividad de parte de los alumnos dado su nivel de complejidad. Si
un alumno opta no hacerlos siempre puede llegar a 10 como nota, pero si opta hacerlos
puede llegar a tener 11 de nota.
Evaluación diferida:
Si por alguna razón, un estudiante no puede realizar una de las evaluaciones anteriormente
detalladas, este podrá realizarla de forma diferida con la misma ponderación del original,
siempre y cuando presente una causa que justique su ausencia; caso contrario podrá
realizarlo con una máxima calicación posible de 8.0. Considerando además que por la
naturaleza del curso, esto no será aplicable para la ultima evaluación.
Los porcentajes de cada una de las actividades se detallan acontinuación:

ACTIVIDAD PONDERACIÓN

Tareas 30 %
Cortos 40 %
Examen semanal 30 %
Créditos 10 %
TOTAL 110 %

Los porcentajes detallados anteriormente consisten de las actividades a realizarse en cada sem-
ana y dependiendo del ritmo de trabajo de los estudiantes el número de cortos y tareas podría
variar, se preveen cuatro cortos y cuatro tareas cada semana.

4
5

7. Jornalización.

Los horarios para clases, recesos, cortos y tareas se detallan a continuación:


HORA ACTIVIDAD
1:30 pm - 2:45 pm Clase.
2:45 pm - 3:15 pm Examen corto.
3:15 pm - 3:45 pm Receso.
3:45 pm - 5:25 pm Clase.
5:25 pm - 5:30 pm Asignación de tarea.

8. BIBLIOGRAFÍA.

o Teoría Combinatoria. Módulo 3. Postgrado para profesores de educación media


o Maria L. Pérez seguí Combinatoria. Cuaderno de olimpiadas matemáticas.
o Pablo S. Bravo. Combinatoria para olimpiadas. Cuaderno de olimpiadas matemáticas.
o Chen Chuan-Chong and Koh Khee-meng. Principles and techniques in combinatorics.
National University of Singapore.
o Jiri Herman, Radan Kucera and Jaromir Simsa. Counting and congurations: Problemas
in Combinatorics, Arithmetic and Geometry..
o Ivan Niven. Matemática de las Opciones o Cómo contar sin contar. Universidad de Oregon.
o María Elena Becker, Norma Pietrocola y Carlos Sanchez. Notas de Combinatoria. Red
Olímpica.
o Titu Andreescu and Zuming Feng. A path to combinatorics for undergraduates. counting
strategies.
o www.artofproblemsolving.com
o www.oma.org.ar

Vous aimerez peut-être aussi